Table des matières

publicité
SP-LP-PT
12/05/09
16:55
Page 1
COLLECTION DURANDEAU
2
de
PROFESSIONNELLE
SCIENCES
BAC PRO
PHYSIQUES ET CHIMIQUES
Livre du professeur
J-P. Durandeau
J-L. Berducou
J-C. Larrieu-Lacoste
C. Mazeyrie
C. Raynal
J-C. Trillaud
Table des matières
1 Quelle différence entre température et chaleur ? CME1 . . . . . . . . . . . . . . . . . . . . . . . . . . . . . . . . . . 4
2 Quels courants électriques dans la maison ou l’entreprise ? CME2 . . . . . . . . . . . . . . . . . . . . . . . . 9
3 Comment protéger une installation électrique ? CME2 . . . . . . . . . . . . . . . . . . . . . . . . . . . . . . . . . . 11
4 Comment évaluer sa consommation d’énergie électrique ? CME2 . . . . . . . . . . . . . . . . . . . . . . . . 16
5 Tous les sons sont-ils audibles ? CME3 . . . . . . . . . . . . . . . . . . . . . . . . . . . . . . . . . . . . . . . . . . . . . . . . 21
6 Comment isoler une pièce du bruit ? CME3 . . . . . . . . . . . . . . . . . . . . . . . . . . . . . . . . . . . . . . . . . . . . 23
7 Comment éviter le basculement d’un corps ? HS1 . . . . . . . . . . . . . . . . . . . . . . . . . . . . . . . . . . . . . . 27
8 Comment soulever un objet ? HS1 . . . . . . . . . . . . . . . . . . . . . . . . . . . . . . . . . . . . . . . . . . . . . . . . . . . . 30
9 Comment soulever facilement un objet ? HS1 . . . . . . . . . . . . . . . . . . . . . . . . . . . . . . . . . . . . . . . . . . 35
10 Quels sont les composants d’une eau minérale ? HS2 . . . . . . . . . . . . . . . . . . . . . . . . . . . . . . . . . . . 38
11 Comment déterminer la concentration d’une substance dans une solution ? HS2 . . . . . . . . . 42
12 Que contient un soda ? HS2 . . . . . . . . . . . . . . . . . . . . . . . . . . . . . . . . . . . . . . . . . . . . . . . . . . . . . . . . . . 46
13 Que se passe-t-il lorsque des produits d’usage courant réagissent entre eux ? HS2 . . . . . . . . . . 50
14 Comment préserver son audition ? HS3 . . . . . . . . . . . . . . . . . . . . . . . . . . . . . . . . . . . . . . . . . . . . . . . 54
15 Comment peut-on décrire le mouvement d’un véhicule ? T1 . . . . . . . . . . . . . . . . . . . . . . . . . . . . 57
16 Comment passer de la vitesse des roues à celle de la voiture ? T2 . . . . . . . . . . . . . . . . . . . . . . . . . 61
Couverture : npeg.fr
Maquette et mise en page : Nicolas Balbo
© HACHETTE LIVRE 2009, 43, quai de Grenelle, 75905 Paris Cedex 15
ISBN 978-2-01-180538-6
www.hachette-education.com
Tous droits de traduction, de reproduction et d’adaptation réservés pour tous pays.
Le Code de la propriété intellectuelle n’autorisant, aux termes de l’article L. 122-4 et L. 122-5, d’une part, que les « copies
ou reproductions strictement réservées à l’usage privé du copiste et non destinées à une utilisation collective », et, d’autre
part, que « les analyses et courtes citations » dans un but d’exemple et d’illustration, « toute représentation ou reproduction
intégrale ou partielle, faite sans le consentement de l’auteur ou de ses ayants cause, est illicite ».
Cette représentation ou reproduction, par quelque procédé que ce soit, sans autorisation de l’éditeur ou du centre français de
l’exploitation du droit de copie (20, rue des Grands-Augustins, 75006 Paris), constituerait donc une contrefaçon sanctionnée
par les Articles 425 et suivants du Code pénal.
Avant-propos
Le livre du professeur, qui accompagne le manuel de
l’élève, comporte les corrigés des activités et des exercices proposés dans chaque chapitre.
Les activités dans le manuel
Nous n’avons pas choisi une méthode déductive
fondée sur la présentation d’un cours préalable suivi
d’exercices d’application, car le dogmatisme qu’elle
peut engendrer nous paraît inadapté dans des classes de Seconde Professionnelle. Nous avons opté au
contraire pour une démarche inductive s’appuyant
sur des activités, le plus souvent expérimentales,
afin de résoudre chaque fois un problème scientifique. Cela permet de mieux impliquer l’élève dans la
construction de ses connaissances, le professeur l’accompagnant, plutôt que le précédant. Le cours (l’essentiel) vient ensuite synthétiser les connaissances et
apporter les compléments nécessaires sur des sujets
qui n’ont pu être abordés dans les activités.
Nous avons formulé, dans le manuel, des questions
pour chaque activité ; l’élève, en répondant à ces
questions, propose des conclusions à la suite de ses
observations et mesures. Le professeur peut alors
aider à reformuler les résultats qui figurent dans le
cours présent dans le manuel.
Ces questions jouant un rôle très important dans la
démarche, nous proposons les réponses dans le livre
du professeur.
Pour aider l’enseignant dans sa tâche :
Le Cédérom met à la disposition de l’enseignant,
pour chaque chapitre :
- une photothèque contenant les images (schémas et photographies) principales du manuel : page
d’ouverture, documents sur les activités, le cours, les
pages de documentation et les exercices ;
- des animations et des vidéos expliquant les notions
essentielles, suivies de fiches élève (questionnaires)
et de fiches enseignant ;
- des schémas bilan animés présentant succinctement l’essentiel du cours ;
- un exercice différencié à deux niveaux.
Le Livre numérique permet de projeter en permanence tous les documents nécessaires à la mise en
œuvre et à l’exploitation du manuel de l’élève.
Les exercices
Dans le livre du professeur, nous fournissons les solutions des exercices figurant dans les cinq rubriques
du manuel de l’élève.
• Tester ses connaissances
Cette forme de présentation et d’évaluation (Q.C.M.)
se répand de plus en plus et présente l’avantage de
développer l’esprit critique. Cette rubrique permet à
chaque élève de tester rapidement les connaissances
fondamentales du chapitre.
• Tester ses capacités
Ces exercices simples traitent des capacités énoncées
dans le référentiel. Chaque concernant une seule
capacité, l’élève peut en vérifier plus facilement
l­’acquisition.
• Appliquer le cours
L’objectif recherché est maintenant plus ambitieux :
il s’agit de vérifier l’aptitude des élèves à appliquer les
lois du programme dans un contexte proche de celui
des activités et de l’essentiel du cours du chapitre.
La résolution de ces exercices nécessite l’acquisition
de savoir-faire dans les domaines théorique et expérimental.
• Utiliser ses connaissances
Les exercices concernés relèvent d’un niveau de difficulté supérieur : il s’agit d’appliquer ses connaissances et ses savoir-faire dans des contextes différents de
ceux des activités et de l’essentiel du cours, en particulier dans la vie professionnelle. Nous avons intégré
dans cette rubrique des exercices dont le thème est le
« développement durable ».
• Situation problème
Ces exercices demandent une plus grande autonomie et davantage de recherche : le travail en groupes
se prête particulièrement à la résolution de ces exercices.
Le livre du professeur constitue un outil indissociable
du manuel, au service des collègues pour le bénéfice
de leurs élèves. Nous acceptons bien volontiers leurs
suggestions et critiques.
1 Quelle différence entre température
et chaleur ? CME1
Les activités
■■Activité 1Comment fabriquer un thermomètre pour mesurer
la température d’un corps ?
•Matériel
Un système de chauffage (chauffe-ballon, ou plaque chauffante, ou bec bunsen)
Un ballon de 250 mL
Une thermistance CTN reliée à un ohmmètre
Des fils de connexion
Un thermomètre électronique
• Produits
Eau et glaçons
* Représentation graphique de la température en fonction de la résistance
de la thermistance utilisée
Température (°C)
100
50
10
0
Résistance (Ω)
10
50
100
•Réponse aux questions
1. Pour la thermistance utilisée :
À 0 °C, R(0) = 330 Ω ; à 100 °C, R(100) = 12 Ω.
2. La courbe est la représentation graphique d’une fonction : en effet, à une valeur de la résistance R
correspond une seule valeur de la température θ.
3. C’est une CTN, car la résistance diminue lorsque la température augmente (pour une CTP, la
résistance augmente avec la température, dans la zone d’utilisation).
4. On mesure la température d’un corps avec un thermomètre : dans cette activité, on a fabriqué et
utilisé un thermomètre électronique dont le capteur est une thermistance CTN.
4 • CHAPITRE 1 - Quelle différence entre température et chaleur ?
■■Activité 2Comment varie la température d’un corps
qui échange de la chaleur ?
•Matériel
Un système de chauffage (chauffe-ballon, ou plaque chauffante, ou bec bunsen)
Un bécher de 100 mL
Une thermistance CTN reliée à un ohmmètre
Des fils de connexion
Un thermomètre électronique
Un chronomètre
•Produits
Eau et glaçons
Expérience 1
Lors du chauffage de l’eau, sa température augmente tout au long du chauffage (par exemple de
18 °C à 72 °C).
Lors du refroidissement de l’eau, sa température diminue tout au long du refroidissement (par
exemple de 52 °C à 27 °C). Exemple ci-dessous :
Température (°C)
50
10
t (min)
0
10
Expérience 2
Tableau des résultats obtenus :
t (min)
État de
l’eau
S : solide
L : liquide
θ (°C)
0
1
2
3
4
5
6
7
8
S
S
S
S
S
S
S
S
S
–1
–1
-0,5
0
0
0
0
0
0
9
10
11
12
S + L S + L S + L S + L
0,5
1
1
1
13
L
1,5
• Réponse aux questions
Expérience 1
1. Lorsqu’on chauffe l’eau, sa température augmente. Lorsqu’on la refroidit, sa température diminue.
2. Lorsqu’on chauffe l’eau, le milieu extérieur cède la chaleur et l’eau la reçoit.
Lorsqu’on refroidit l’eau, l’eau cède la chaleur au milieu extérieur.
CHAPITRE 1 - Quelle différence entre température et chaleur ? • 5
3. Un corps froid peut fournir de la chaleur à un autre corps si la température de celui-ci est plus
basse que la sienne (c’est le cas dans la seconde expérience).
Expérience 2
4. Lorsque la glace fond, sa température est constante.
5. Le changement d’état observé est la fusion de la glace.
6. La chaleur reçue par la glace provient de l’eau froide.
Les documents
■■La maison passive
Réponse aux questions
1. Pour une maison de surface habitable 120 m2, neuve et classique (donc avec une bonne isolation),
l’énergie totale nécessaire annuellement est :
110 × 120 = 13 200 kWh.
2. Dans une VMC « double flux », l’air vicié expulsé de la maison (premier flux) est chaud et il cède de
la chaleur à l’air neuf entrant (second flux) qui est à une température plus basse.
Un puits canadien ou provençal est une canalisation contenant un fluide, constituant un réseau soit
dans le sol à une certaine profondeur, soit dans une nappe d’eau souterraine : le fluide circulant dans
ce réseau reçoit de la chaleur provenant de la terre ou de l’eau et en cède une partie à l’intérieur de la
maison.
3. Un pont thermique est une rupture d’isolation thermique entre deux éléments d’un bâtiment dont
les résistances thermiques sont différentes.
Exemples :
- à l’angle entre deux parois verticales ;
- entre une paroi verticale et une paroi horizontale ;
- au joint entre un mur et un châssis de fenêtre.
■■La pompe à chaleur géothermique
Réponse aux questions
1. L’installation géothermique décrite comprend trois circuits de fluides :
- le circuit de captage dans le sol ;
- le circuit frigorigène de la pompe à chaleur ;
- le circuit de chauffage de l’habitation.
2. Le fluide frigorigène récupère la chaleur amenée par le fluide du circuit de captage, à 10 °C environ.
3. Le fluide frigorigène cède la chaleur accumulée au fluide du circuit de chauffage,
à 70 °C environ dans notre exemple.
4. Dans l’évaporateur, le fluide frigorigène capte de la chaleur provenant du fluide du circuit de
captage et voit sa température augmenter de 0 °C à 5 °C : le fluide frigorigène passe de l’état liquide à
l’état gazeux.
Dans le condenseur, la température du fluide frigorigène passe de 70 °C à 32 °C : il cède de la chaleur
au fluide du circuit de chauffage de l’habitation. Le fluide frigorigène passe de l’état gazeux à l’état
liquide.
6 • CHAPITRE 1 - Quelle différence entre température et chaleur ?
Les exercices
Tester ses connaissances
b. La température de solidification de l’eau pure est
de 0 °C.
Q.C.M.
1 : B 2 : A et C 3 : C 4 : A 5 : A et B
c. L’eau salée ne se solidifie pas complètement à une
température constante (la courbe ne montre pas de
palier).
Tester ses capacités
9. Expérience
1. Lire un thermomètre
1 : 17,5 °C 2 : 68,0 °F 3 : –6 °C 4 : 37 °C.
2. Lire une courbe
a. Le palier se forme à 42 °C environ : il correspond à
un changement d’état du corps.
b. Au début de l’expérience, le corps est liquide ; au
bout de 9 minutes, il est solide.
c. Le changement d’état observé est une solidification.
d. Le corps est de la paraffine, car la température de
solidification de l’eau pure est de 0 °C.
Appliquer le cours
3. Lire un thermomètre à alcool
Températures lues : 22 °C ; 21 °C ; 20 °C.
La lecture correcte est : 21 °C.
4. Lectures de températures
Température mesurées : 39 °C ; –6 °C ; –6 °C.
5. Température de la pièce
a. Les unités : ° F  degré Fahrenheit et °C  degré
Celsius.
b. La température du salon est de 19,5 °C.
Expérience 1 : on a réchauffé le corps ; le palier observé
correspond à la vaporisation de l’eau.
Le corps est bien de l’eau pure.
Expérience 2 : on a refroidi le corps ; le palier observé
correspond à une solidification.
Le corps est de la paraffine.
10. Grandeurs et unités
Lettre
représentant
la grandeur
Unité de la
grandeur
Symbole
de
l’unité
Quantité de
chaleur
Q
joule
J
Masse du
corps
m
kilogramme
kg
Capacité
thermique
massique
c
joule par
kilogramme
par degré
Celsius
J/(kg.°C)
Température
initiale
θi
degré Celsius
°C
Température
finale
θf
degré Celsius
°C
Nom de la
grandeur
11. Expérience
6. Changement d’échelle
a. L’eau du verre B reçoit de la chaleur : sa tempéra-
a. 0 °C  273 K ; 100 °C  373 K.
b. –40 °C  233 K ; 83 °C  356 K.
c. 100 K  –173 °C ; 293 K  20 °C.
ture augmente.
b. L’eau du verre A cède de la chaleur au glaçon : sa
température diminue.
7. Fahrenheit
a. À un écart de 100 °C correspond un écart de 180 °F ;
12. Bouilloire électrique
à un écart de 1 °C correspond un écart de 1,8 °F.
b. À la température d’un appartement de 20 °C correspond :
32 + 20 × 1,8 = 68 °F.
c. À la température du corps humain à 98,6 °F correspond :
98,6 – 32 = 66,6.
66,6 ÷ 1,8 = 37 donc 37 °C.
a. Quantité de chaleur reçue par l’eau de la
8. Courbes de refroidissement
par l’eau au milieu extérieur.
a. La courbe 2 est celle de l’eau pure, car elle possède
b. Lorsque 200 g d’eau se vaporisent à 100 °C, il faut
un « palier » à 0 °C.
leur fournir : 2 × 226 = 452 kJ.
bouilloire :
Q = m.c (θf – θi) = 0,5 × 4 180 × (80 – 20) = 125 400 J.
b. La chaleur reçue par l’eau provient de la résistance
électrique de la bouilloire.
13. Changement d’état et chaleur
a. Il échange aussi 335 kJ : cette chaleur est fournie
CHAPITRE 1 - Quelle différence entre température et chaleur ? • 7
Utiliser ses connaissances
18. Fabrication de potences VTT
14. Chaleur latente
Changement
d’état
Chaleur
latente
massique L
Masse
Fusion
335 kJ/kg
1 kg
335 kJ
Chaleur
reçue
Solidification
–335 kJ/kg
200 kg
–67 000 
kJ
Chaleur
cédée
Vaporisation
2 260 kJ/kg
1 kg
2 260 kJ
Chaleur
reçue
Liquéfaction
–2 260 kJ/kg
500 g
–1 130 kJ
Chaleur
cédée
Q (kJ)
Chaleur
cédée ou
chaleur
reçue
15. Choisir un thermomètre
a. En France, on utilisera un thermomètre à alcool,
car [– 35 °C ; 42 °C] est inclus dans l’intervalle
[-117 °C ; 78 °C].
b. Pour repérer la température d’ébullition de l’eau
pure (100 °C), on utilisera un thermomètre électronique, car 100 appartient à l’intervalle [0 ; 200].
16. Maison passive (Développement durable)
a. La maison passive réalise l’essentiel de ses économies dans le chauffage de la maison.
b. La consommation d’énergie pour chaque type de
maison :
Type
Maison existante
Nouvelle construction
Maison basse énergie
Maison passive
Énergie pour une
surface habitable de
110 m2
270 × 110 = 
29 700 kWh
170 × 110 = 
18 700 kWh
110 × 110 = 
12 100 kWh
45 × 110 = 
4 950 kWh
17. La fonte d’un névé (Développement
durable)
a. Quantité de chaleur nécessaire à la fonte du
névé :
Q = m.L = 30 000 × 335 000 = 1 × 1010 J
b. En 1 heure, le névé reçoit :
130 × 40 × 1 = 5 200 Wh soit 5,2 kWh
ou 5,2 × 3,6 × 106 = 1,87 × 107 J.
D’où la durée de la fonte : 1 × 1 010 ÷ 1,87 × 107
≈ 535 heures soit 540 h à la dizaine d’heures près.
a. Pour le porter à sa température de fusion :
Q = m.c (θf – θi) = 0,4 × 900 ×(660 – 25) = 228 600 J.
b. Pour fondre 0,4 kilogramme d’aluminium à la
température de 660 °C :
Q = m.L = 0,4 × 404 000 = 161 600 J.
c. Lors de sa fusion, la température de l’aluminium
reste constante.
19. Je recherche sur le Net
Anders Celsius (1701-1744) : professeur d’astronomie
à Uppsala en Suède, il construit un thermomètre pour
ses recherches en météorologie. Il étudie les aurores
boréales, les caractéristiques de la Terre,….
Daniel Gabriel Fahrenheit (1686-1736) : physicien
allemand inventeur de l’aréomètre (permet de mesurer des densités) et du thermomètre à mercure qui
portent son nom.
a. Celsius avait fixé la première échelle : 0 : température d’ébullition de l’eau et 100 : température de
congélation de la glace : Il exprimait ses mesures en
degré centigrade.
Son échelle a donc été inversée.
b. Fahrenheit a fixé la graduation 0 de son échelle à
la température la plus basse dans sa ville de Dantzig
lors de l’hiver 1708/1709.
La graduation 96 correspondait à la température du
sang d’un cheval.
20. Eau fraîche en Afrique (Développement
durable)
La pellicule d’eau à la surface de la jarre se vaporise :
le milieu extérieur à cette pellicule doit donc lui fournir de la chaleur. En particulier l’eau de la jarre qui
voit sa température baisser en fournissant de la chaleur à la pellicule d’eau. Ainsi, l’eau contenue dans la
jarre reste fraîche.
21. Solidification d’un corps pur
a. Première étape : faire fondre de la paraffine dans
un tube à essai.
Seconde étape : plonger le tube de paraffine dans un
bécher d’eau + glace.
b. Mesurer la température de l’eau dans le bécher.
On observe le changement d’état de la paraffine.
On compare les températures initiale et finale (quand
la paraffine est à l’état solide) de l’eau dans le bécher.
L’élévation de cette température montre que la paraffine a cédé de la chaleur au milieu extérieur lors de sa
solidification.
8 • CHAPITRE 1 - Quelle différence entre température et chaleur ?
2 Quels courants électriques dans la maison
ou l’entreprise ? CME2
Les activités
■■Activité 1Les tensions à la sortie de l’adaptateur d’un téléphone
portable et de la prise de courant du secteur sont-elles
semblables ?
•Matériel
Un ordinateur + interface EXAO ou un oscilloscope
Un adaptateur de téléphone portable
Un générateur de courant alternatif 6V/12V
Des fils de connexion
•Expérience 2
Les deux courbes obtenues ressemblent à celles du doc 2.
Dans le cas d’un chargeur de téléphone portable, on peut aussi enregistrer une tension totalement
redressée mais non lissée.
•Réponse aux questions
1. Aux bornes du générateur, la tension observée est alternative (sinusoïdale) donc variable au cours
du temps.
Aux bornes de l’adaptateur, la tension observée est constante.
2. Le courant du secteur est alternatif ; il est différent du courant continu fourni par un adaptateur de
téléphone portable.
■■Activité 2 Quelles sont les caractéristiques d’une tension alternative ?
•Matériel
Un ordinateur + interface EXAO ou un oscilloscope
Un multimètre
Un générateur très basse fréquence GTBF
Un fréquencemètre (pas obligatoire)
Des fils de connexion
•Expérience
La courbe analysée ressemble à celle du doc 4.
•Réponse aux questions :
1. Dans notre exemple, on dénombre 5 motifs élémentaires, donc 5 périodes en une seconde.
La fréquence était réglée exactement sur 5 hertz.
Le nombre de périodes par seconde est donc égal à la fréquence.
1
2. Formule : f = avec T en seconde et f en hertz.
T
3. Dans notre exemple, numériquement, Umax = 10 V ; U = 7 V.
U
U
On vérifie que : m ax = 2 et m ax = U .
U
2
CHAPITRE 2 - Quels courants électriques dans la maison ou l’entreprise ? • 9
Les exercices
Tester ses connaissances
Q.C.M.
1 : A et C
5 : B
Appliquer le cours
11. Écran d’EXAO
2 : A et C
6 : A et B
3 : A, B et C
7 : B et C
4 : B et C
8 : B et C.
Tester ses capacités
1. Corriger
a. Une tension alternative est tantôt positive, tantôt
négative.
b. Une tension continue est constante au cours du
temps.
c. Un ordinateur muni d’une interface EXAO (ou
un oscilloscope) permet de visualiser les variations
d’une tension au cours du temps.
a. Le signal est sinusoïdal.
b. T = 20 ms ; f = 50 Hz. C’est la même fréquence que
celle du réseau.
c. Umax = 9 V . U =
U max
2
=
9
2
≈ 6, 4 V .
12. Prise du secteur
a. Schéma :
Protection électrique
………………………………….....
Phase
………………………………….....
Neutre
2. Observer
………………………………….....
Tension continue : b ;
Tension alternative : a, c et d.
3. Questionnaire
a. Une tension est alternative si sa valeur change de
signe.
b. Une tension est périodique si elle se reproduit
régulièrement de manière identique.
4. Analyse de tensions
a. Tension alternative  b et d.
b. Tension périodique  b, c et d.
c. Tension alternative périodique  b et d.
5. Tension maximale et période
a. La tension maximale est de : 3 × 0,5 = 1,5 V.
b. La période est de : 6 × 5 = 30 ms.
6. Motif élémentaire et période
a. Périodes : premier graphe : [ac], deuxième gra-
b. La période du secteur est de 20 ms.
c. La valeur maximale de la tension est de 325 V.
13. Mesures et calculs
a et b
Symbole
Grandeur
Appareil de mesure
U
Tension efficace
Voltmètre
Umax
Tension
maximale
Oscilloscope ou
système EXAO
T
Période
Oscilloscope ou
système EXAO
f
Fréquence
Fréquencemètre
c. Si f = 50 Hz alors T = 20 ms et non 25 ms.
Autre raisonnement : si T = 25 ms alors f = 40 Hz.
14. Lire une façade d’oscilloscope
a. Durée de balayage : 5 ms/DIV ; une période occupe
phe : [ab], troisième graphe : [ae].
b. Valeurs : ac = 10 ms ; ab = 50 ms ; ae = 4 s.
4 DIV d’où T = 4 × 5 = 20 ms.
b. Sensibilité verticale : 5 V/DIV ; la hauteur d’un pic
occupe 3 DIV d’où Umax = 3 × 5 = 15 V.
7. Lire une période
15. Lecture d’écran
T = 8 s ; T = 20 ms.
8. Lire une amplitude
a. Amplitudes : 8 V ; 30 V.
b. On la nomme aussi « tension maximale ».
1
1
=
= 250 Hz .
T 0,004
12,5
b. Umax = 2,5 × 5 = 12,5 V d’où U =
≈ 8,8 V .
2
a. T = 2 × 2 = 4 ms d’où f =
9. La tension du secteur
Utiliser ses connaissances
a. U max = U . 2 = 230 × 2 ≈ 325 V .
16. Au bureau
1 1
b. T = =
= 0, 02 s ou 20 ms.
f 50
a. La tension d’entrée est alternative sinusoïdale de
10. Tracé d’une sinusoïde
Période : T =
1
1
=
= 100 s .
f 0, 01
tension efficace 240 V au plus.
L’intensité est inférieure ou égale à 1,5 A. La plage des
fréquences est de 50 Hz à 60 Hz.
b. La tension de sortie est continue, de valeur 19 V.
10 • CHAPITRE 2 - Quels courants électriques dans la maison ou l’entreprise ?
L’intensité fournie à l’ordinateur est de 4,74 A.
Il n’y a pas de fréquence indiquée, car la tension n’est
pas périodique.
19. Chez le garagiste
17. Sur le chantier, à l’atelier
b. Il y a bien une injection par tour moteur
a. La perceuse sans fil fonctionne sous tension
continue ; cette tension électrique est délivrée par des
accumulateurs rechargeables.
b. La perceuse Dremel fonctionne en courant alternatif ; elle se branche directement sur le secteur.
a. A : 0,5 tour moteur ; B : 0,5 tour moteur ; C : 1 tour
moteur ; D : 1 tour moteur.
(courbe D).
60
= 0,02 s .
3000
Donc les périodes précédentes sont de :
c. A ce régime, un tour moteur dure :
A et B : 10 ms ; C et D : 20 ms.
20. Développement durable : les modules
18. Dans la voiture : le GPS embarqué
a. Le GPS de voiture fonctionne sous tension continue de 5 V, le courant qui l’alimente ayant une intensité de 1 A.
b. Le cordon permet de le brancher sur l’allumecigare (12 V), car il accepte une tension continue
comprise entre 10 V et 30 V ; il supporte une intensité
jusqu’à 1,3 A alors que le fonctionnement du GPS ne
nécessite que 1 A.
photovoltaïques
a. Le générateur fournit un courant continu.
b. L’onduleur convertit le courant continu en courant alternatif.
La batterie accumule l’énergie électrique reçue sous
forme d’énergie chimique.
c. Un adaptateur de téléphone portable sera branché sous tension alternative, donc sur l’onduleur.
3 Comment protéger
une installation électrique ? CME2
Les activités
■■Activité 1 Quel est le rôle d’un coupe-circuit ?
•Matériel pour un poste
Un générateur à courant alternatif 6 V
4 lampes 6 V/ 0,6 W
4 interrupteurs
Des fils de connexion
Un fusible de 250 mA type F rapide sur son porte fusible
Un ampèremètre
Une platine
•Réponses aux questions
1. L’intensité du courant principal augmente avec le nombre de lampes sous tension.
La loi d’additivité des intensités de courant pour un montage en dérivation justifie ce phénomène.
2. L’ouverture du circuit n’est pas instantanée lorsque l’intensité dépasse 250 mA.
Le temps de fusion dépend de la surcharge (il sera plus rapide avec quatre lampes qu’avec trois).
3. En connectant le point A au point B (bornes de chaque branche dérivée), on crée un court-circuit.
Le fusible de l’ampèremètre et/ou celui du porte-fusible fondent pour provoquer l’ouverture du
circuit.
4. Un coupe-circuit protège les appareils en intervenant lors des courts-circuits ou des surcharges.
Il ouvre automatiquement le circuit lorsque l’intensité du courant dépasse pendant un temps
déterminé la valeur indiquée par leur calibre.
CHAPITRE 3 - Comment protéger une installation électrique ? • 11
■■Activité 2À quelle condition un disjoncteur différentiel protège-t-il
les personnes ?
•Matériel pour un poste
Un générateur 12 V
Une maquette (câblée sur le schéma TT) comprenant :
- un disjoncteur différentiel ;
- un module machine ;
- une prise trois bornes ;
- un mannequin équipé d’une DEL qui s’allume lorsqu’il est électrisé Des fils de connexion
•Réponses aux questions
1. La DEL du mannequin s’allume pour signaler l’électrisation dans l’expérience 1 : la carcasse de la
machine n’est pas reliée à la terre.
2. Doc 2 : le courant effectue le parcours : fil de phase – carcasse métallique du lave-linge –
mannequin (la DEL éclaire, il est électrisé) – fil de mise à la terre par EDF.
Doc 3 : le courant effectue le parcours : fil de phase – carcasse du lave-linge – fil de protection
électrique (mise à la terre du lave-linge) – fil de mise à la terre par EDF.
Le mannequin n’est pas électrisé.
3. Le conducteur différentiel protège les personnes lorsque les carcasses métalliques des machines
sont reliées à la terre par un conducteur de protection électrique.
Les documents
■■Une installation domestique
•Réponses aux questions
1. - Le circuit des prises comporte trop de prises.
- Même remarque pour le circuit des lampes. De plus, le calibre du fusible est trop faible.
- Le chauffe-eau, le congélateur, la cuisinière électrique doivent avoir un circuit par appareil.
2. La nouvelle installation à usage similaire :
Disjoncteur de
branchement
de 500 mA
Disjoncteur
de 30 mA
1,5 mm2
5 lampes
5 lampes
10 A
1,5 mm2
10 A
16 A
6 mm2
32 A
20 A
1,5 mm2
10 A
5
prises
cuisinère
congélateur
2,5 mm2
Ph
N
1,5 mm2
2,5 mm2
10 A
5
prises
Chauffeeau
16 A
PE
radiateur
Les dangers de la tension monophasée du secteur
•Réponses aux questions
1. Il y a contact direct lorsqu’une personne entre en contact avec les parties actives des matériels
électriques (soit les conducteurs actifs de la phase ou du neutre, soit les pièces conductrices se
trouvant sous tension en service normal : vis, barrettes,…).
12 • CHAPITRE 3 - Comment protéger une installation électrique ?
Il y a contact indirect lorsqu’une personne entre en contact avec la masse métallique d’un appareil
mise accidentellement sous tension à la suite d’un défaut d’isolement.
2. Le contact direct des deux fils ne produira pas de courant de défaut, car le corps est isolé
électriquement de la terre par un tapis isolant. Sans mise à la terre, le disjoncteur différentiel ne
détecte pas le défaut. Le courant électrisera la personne.
3. Dans le cas 3, l’oiseau n’est pas électrisé, car il n’est pas soumis à une tension de contact
(Uc ≈ 0 V).
Les exercices
Tester ses connaissances
Interrupteurs
ouverts
K2,
K3
K1,
K3
K1,
K2
K3
K2
aucun
Q.C.M.
Interrupteurs
fermés
K1
K2
K3
K1,
K2
K1
K3
K1,K2,
K3
Intensité du
courant I (A)
0,17
0,26
0,43
0,43
0,60
0,86
1 : A et C 2 : A et C
3 : A, B et C 4 : B et C
Tester ses capacités
1. Protéger du matériel
- Un circuit d’éclairage comprenant 8 lampes se
protège avec un disjoncteur divisionnaire de 16 A ;
- un circuit de 5 prises de courant se protège avec
un disjoncteur divisionnaire de 16 A ;
- un circuit de cuisinière électrique se protège avec
un disjoncteur divisionnaire de 40 A.
Appliquer le cours
5. Défauts électriques
Les défauts électriques peuvent provoquer des surtensions, des surintensités, des courts-circuits ou des
courants de défaut.
a. Les surtensions augmentent la tension U (volt)
aux bornes des appareils (foudre).
2. Deux appareils de protection
a. Le calibre de 20 A indique que le coupe-circuit est
capable d’interrompre le courant en ouvrant automatiquement le circuit lorsque celui-ci dépasse 20 A.
Cette coupure intervient au bout d’un certain temps.
b. Leur fonction commune est de protéger le matériel installé en cas de surintensité (dépassement du
courant nominal) ou de court-circuit. Les deux appareils les détectent et coupent le circuit.
c. Le fusible intervient par fusion en cas de surcharge. On doit changer la cartouche avant de remettre la ligne sous tension. C’est la différence avec le
disjoncteur divisionnaire qui est muni d’un système
de déclenchement mécanique provoquant l’ouverture du circuit en cas de surcharge. On le réenclenche
pour remettre la ligne sous tension.
Les surintensités et les courts-circuits augmentent
l’intensité I (ampère) du courant dans les conducteurs.
Le courant de défaut Id modifie les valeurs des intensités des courants dans le conducteur de phase Iph et
le conducteur du neutre In.
On a Id = Iph – In ; Id s’exprime en ampère.
b. Exemples : surtension : la foudre ; surintensité :
trop d’appareils utilisant des intensités importantes
sur une multiprise ; court-circuit : contact de deux
conducteurs dénudés accidentellement ; courant de
défaut : usure d’une gaine de conducteur de phase au
contact d’une carcasse métallique de machine.
6. Appareils de protection
(Dans les deux cas, la fermeture du circuit ne se fait
qu’après avoir réparé le défaut électrique qui est la
cause du déclenchement.)
a. - Les parafoudres protègent des surtensions.
3. Multiprise
- Les disjoncteurs différentiels associés à une bonne
prise de terre reliée aux masses des machines protègent des défauts d’isolement.
L’intensité efficace du courant appelé par la multiprise est :
I = 3 + 5 + 1,25 = 9,25 A.
4. Additivité des intensités
Le tableau complété :
- Les fusibles et les disjoncteurs divisionnaires protègent des courts-circuits et des surcharges.
b. Les fusibles et les disjoncteurs divisionnaires protègent les installations. Les disjoncteurs différentiels
associés à une bonne prise de terre protègent les personnes.
CHAPITRE 3 - Comment protéger une installation électrique ? • 13
7. Ouverture d’un circuit
a. Un fusible de 16 A fond lorsqu’il est parcouru pendant un certain temps (voir la courbe activité 1) par
un courant d’intensité supérieure à 16 A.
b. Un disjoncteur différentiel de sensibilité 30 mA
se déclenche avant que l’intensité d’un courant de
défaut atteigne 30 mA. (Sa plage de déclenchement
va de 15 mA à 30 mA).
c. Un disjoncteur divisionnaire, calibre 20 A, se
déclenche lorsque la valeur de l’intensité efficace du
courant dans la ligne atteint 20 A.
d. Le disjoncteur général de branchement de calibre
60 A et de sensibilité 500 mA se déclenche :
- lorsque l’intensité efficace du courant appelée par
l’installation qu’il protège est supérieure à 60 A (elle
correspond à la valeur souscrite à l’abonnement) ;
- avant que l’intensité d’un courant de défaut n’atteigne 500 mA (sa plage de déclenchement va de
250 mA à 500 mA).
8. Tableau électrique
prise risque de provoquer un coupe-circuit, car elle
est prévue pour admettre un courant d’intensité efficace inférieure à 16 A.
11. Circuit « prises de courant »
a. Le fusible à cartouche de 16 A protège une ligne
admettant 8 points d’utilisation avec une section de
conducteur en cuivre de 2,5 mm².
b. Les trois appareils sont des conducteurs ohmiques : ils convertissent l’énergie électrique en chaleur.
c. La puissance transférée à ces trois appareils est :
P = 600 + 800 + 1 500 = 2 900 W.
Lorsqu’ils fonctionnent ensemble, l’intensité du courant dans le circuit principal est :
2 900
= 12,6 A.
230
Le fusible de 16 A n’ouvrira pas le circuit : il protège
le circuit.
I = 
12. Risque d’électrisation
a. Seule une personne habilitée peut intervenir sur
un tableau alimenté par la tension du secteur.
b. Les dénombrer et les identifier sur le tableau ; il y a
autant de coupe-circuits que de lignes.
c. Les circuits alimentant les appareils sont séparés
afin :
- de limiter l’intensité efficace dans les conducteurs
de ligne ;
- de ne pas mettre hors tension toute une installation en cas de défaut électrique sur un appareil.
d. Le circuit appelant la plus grande intensité est
protégé par le coupe-circuit de plus grand calibre.
Contact direct
Contact direct
9. Une installation
a. L’intensité efficace traversant L2 est :
I2 = 0,85 – 0,3 – 0,25 = 0,30 A.
b. La puissance transférée par chacune des lampes
est :
P1 = P2 = 240 × 0,30 = 72 W.
10. Une plaque signalétique
a. La plus grande intensité efficace admise par ce
bloc prise est :
P 3680
=
= 16 A.
U 230
b. L’intensité efficace appelée par les appareils
lorsqu’ils fonctionnent est :
I = 6,5 + 5,2 + 7,1 = 18,8 A.
Cette intensité efficace dépasse l’intensité nominale
prévue par la construction ; donc on n’est pas dans
la norme.
c. Si les trois appareils fonctionnent simultanément
(I = 18,8 A), le fusible ne coupera pas le circuit (calibre
20 A). L’échauffement des conducteurs de la multiI = 
Contact indirect
13. Un enrouleur de rallonge
a. La section de chaque conducteur est de 1,5 mm².
b. Lorsque le câble est enroulé, la puissance nominale que peut transmettre l’enrouleur est de 1 200 W.
Lorsque le fil est déroulé, cette puissance est de
3 500 W.
P 3 500
c. P = U.I donc I =  =
= 15,2 A.
U
230
d. Lorsque le câble est déroulé, on peut utiliser cette
rallonge, mais le fusible de 10 A ne supportera pas
l’intensité efficace de la charge, car 10 A < 15,2 A : le
fusible fond.
14. Bonne ou mauvaise prise de terre ?
a. Ra = 
UL
25
=
= 830 Ω (Le disjoncteur est de
I Δ 30 × 10 −3
haute sensibilité).
14 • CHAPITRE 3 - Comment protéger une installation électrique ?
b. Si le local est sec, on a UL = 50 V, avec I Δ = 500 mA
la valeur de la prise de terre doit être inférieure à :
50
= 100 Ω.
0,5
Une prise de terre de 830 Ω constitue un défaut électrique, car sa résistance est supérieure à 100 Ω (perte
d’efficacité).
15. Défaut de mise à la terre
a. Le tableau complété :
Sensibilité du
différentiel
IΔ
Résistance
mesurée
Ra (Ω)
Résistance maximale
U
Rmax =  L (Ω)
IΔ
3A
15
16,7
1A
40
50
500 mA
150
100
100 mA
400
500
30 mA
550
1 700
b. La prise de terre défectueuse est celle qui est
mesurée par un disjoncteur différentiel de 500 mA,
car elle est supérieure à 100 Ω (150 > 100).
Utiliser ses connaissances
16. Un conducteur défectueux
a. Le régime de neutre à la terre est dit schéma TT.
- Le neutre est directement relié à la terre au niveau
du transformateur de distribution de l’énergie électrique : premier T.
- Les masses d’utilisation sont reliées à une prise
de terre par un conducteur de protection électrique :
deuxième T
La couleur du conducteur de protection électrique
(PE) est jaune et vert.
30 mA : le disjoncteur différentiel ouvre le circuit
obligatoirement dès que la différence des intensités
des courants dans le conducteur de phase et celui du
neutre atteint cette valeur.
b. Voir schéma : la boucle de courant est surlignée en
noir.
Réseau de
distribution
Rn = 25 Ω
Disjoncteur
divisionnaire
c. Calcul du courant de défaut Id :
Id = 
U
230
=
= 1,84 A.
Ra + Rn 125
Avec une sensibilité de 30 mA, le disjoncteur différentiel détecte le défaut et déclenche l’ouverture du
circuit avant qu’une personne entre en contact avec
la carcasse de la machine.
d. Avant de réenclencher le disjoncteur, il faut effectuer les opérations de maintenance nécessaires pour
corriger le défaut.
17. Défaut d’isolement et intensité du courant
a. Une personne parcourue par un courant de 0,2 mA
durant 2 secondes ne court pas de risque.
b. Une personne parcourue par un courant de
0,2 A durant 2 secondes risque de graves brûlures, la
fibrillation ventriculaire et l’arrêt cardiaque.
c. La fibrillation ventriculaire correspond à une série
de contractions violentes et désordonnées du muscle
cardiaque pouvant entraîner la mort par électrocution.
d. Pour éviter le risque de fibrillation ventriculaire,
un courant de 100 mA ne doit pas traverser le corps
humain durant plus d’une seconde.
18. Résistance du corps humain et tension de
contact
a. Lorsque la tension de contact augmente, la résistance électrique du corps humain diminue.
b. Lorsque l’humidité augmente, la résistance électrique du corps humain diminue.
c. Soumise à 250 V, une peau mouillée a une résistance de 800 Ω.
d. Uc = Rc.I .
250
L’intensité du courant est I = 
= 0,31 A.
800
e. Il faut isoler ce circuit en moins de 200 ms pour
éviter l’arrêt cardiaque.
19. Installer une cuisinière électrique
a. La puissance transférée lorsque toutes les plaques
fonctionnent est de 7 000 W et le courant appelé est
7000
de
= 30,5 A. Le fusible fond et ouvre le circuit.
230
b. Pour les plaques de cuisson, les conducteurs de
la ligne d’alimentation doivent avoir une section
de 6 mm² et posséder un coupe-circuit de 32 A ou
40 A. Axel et Julianne doivent tirer une nouvelle ligne
ayant ces caractéristiques.
Ra = 100 Ω
CHAPITRE 3 - Comment protéger une installation électrique ? • 15
4 Comment évaluer sa consommation
d’énergie électrique ? CME2
Les activités
■■Activité 1Comment comparer les consommations d’énergie
de deux lampes ?
•Matériel
Deux lampes montées sur un support : une à incandescence (12 V ; 25 W) et l’autre fluocompacte
(12 V – 7 W)
Un générateur (12 V)
Un joulemètre
Des fils de connexion
•Remarque
On trouve dans le commerce deux types de lampes pouvant fonctionner sous 12 V, ce qui permet une
manipulation sous très basse tension en toute sécurité.
•Réponses aux questions
1. Pour un éclairage identique, c’est la lampe fluocompacte qui mérite la qualification de LBC, car elle
consomme beaucoup moins qu’une lampe à incandescence.
E1
= 3,5, on peut écrire que la lampe fluocompacte consomme 3,5 fois moins d’énergie
E2
électrique que la lampe à incandescence.
2. Si
3. On a
E1 P1
≈
≈ 3,5 . L’énergie consommée est donc proportionnelle à la puissance de la lampe.
E 2 P2
4. L’utilisation de lampes fluocompactes permet d’économiser de l’énergie électrique.
■■Activité 2Quelles sont les grandeurs influant sur l’énergie électrique
transférée à un appareil ?
•Matériel
Un joulemètre Un générateur Une lampe 6V – 6 W ou autre, d’une puissance de quelques watts Un interrupteur Des fils de connexion
•Réponses aux questions
1. L’énergie indiquée par le joulemètre est proportionnelle à la durée de fonctionnement de la lampe.
2. On a P.t ≈ E.
3. L’énergie électrique s’exprime en joule lorsque la puissance P est en watt et la durée de
fonctionnement en seconde.
16 • CHAPITRE 4 - Comment évaluer sa consommation d’énergie électrique ?
Les documents
■■Différents moyens d’éclairage
•Réponses aux questions
La case cochée indique que la caractéristique est vérifiée :
Caractéristiques
Dure le plus
longtemps
Lampes à
incandescence
Est une lampe
basse
consommation
Restitue la plus
grande quantité de
lumière par watt
Rend les couleurs
correctement
✘
fluorescence
✘
DEL
✘
✘
✘
■■Étude comparative de deux installations d’éclairage
•Réponses aux questions
1. Le coût annuel de la consommation est de 39 € (vérification : 150 × 2 × 1 000 × 10-3 × 0,13 = 39) pour
l’installation réalisée avec les lampes halogènes et de 8,97 € (vérification : 23 × 3 × 1 000 × 10-3 × 0,13
= 8,97) pour l’installation réalisée avec les lampes fluocompactes.
2. L’installation réalisée avec les lampes fluocompactes présente un surcoût initial de 25 €.
La différence des coûts annuels de consommation est de : 39 – 8,97 = 30,03 €. Le surcoût initial est
donc amorti au bout de la première année de fonctionnement (25 < 30,03).
3. L’installation la plus économique au bout de cinq ans est celle des lampes fluocompactes ;
l’économie réalisée pendant cette durée est de : 210 – 74,85 = 135,15 soit 135,15 €.
Les exercices
Tester ses connaissances
Q.C.M.
1 : B et C
2 : B et C
3 : C
K
Joulemètre
4 : B et C.
000
Tester ses capacités
U
t
We
mode P
I
U
1. Compteur
a. La grandeur physique indiquée par l’afficheur est
l’énergie électrique consommée.
b. Le kilowattheure est une unité pratique.
2. Mesurer une énergie électrique
Q.C.M.
1 : A et C 2 : B
J
W kJ
A
V
I
départ
arrêt
G
b. À à la fin de l’expérience, le joulemètre indique :
E = P . t = 12 × 200 = 2 400 J soit 2,4 kJ.
3 : C.
3. Énergie pour un cycle
b. L’énergie électrique transférée durant ce cycle est
Appliquer le cours
87 547 – 87 532 = 15 kWh.
c. Par le calcul, on obtient :
E = P .t = 10 × 1,5 = 15 kWh.
5. Conversion et préfixes
4. Joulemètre
144 TJ = 1,44 × 1014 J .
a. En utilisant les préfixes,
- en joule : 36 kJ = 3,6 × 104 J ; 72 MJ = 7,2 × 107 J ;
a. Le schéma du montage (page 47 doc 2 du livre de
- en watt-heure : 5 mWh = 5 × 10-3 Wh ;
l’élève)
8 kWh = 8 × 103 Wh ; 9,2 GWh = 9,2 × 109 Wh.
CHAPITRE 4 - Comment évaluer sa consommation d’énergie électrique ? • 17
b. En utilisant les unités pratiques,
12. Budget TV
-en joule : 3,2 × 103 Wh = 1,152 × 107 J ; 5,2 kWh
= 1,872 × 107 J.
-en watt-heure : 36 MJ = 104 Wh ; 54 000 kJ
= 1,5 × 104 Wh.
a. Le téléviseur a fonctionné pendant t = 
6. Entre deux relevés
a. L’énergie distribuée au cours de cette durée est de
1028 kWh.
b. La durée de la consommation a été de 185 jours.
(19+30+31+30+31+31+13 = 185).
c. La consommation journalière moyenne d’énergie
électrique a été de 5,56 kWh/jour.
d. Le montant de cette consommation journalière
est de : 5,56 × 0,13 = 0,72 €.
= 4 h.
b. L’énergie consommée par le téléviseur de Jules en
un an est : E = 0,52 × 365 ≈ 190 kWh.
c. Le coût de cette consommation d’énergie est de :
0,13 × 190 = 24,70 €.
13. Ordinateur portable
a. Représentation
50
40
30
20
a.
10
11,9 V
1,76 A
100 s
20,8 W
Grandeurs
physiques
tension
intensité
durée
puissance
b. L’afficheur indique en kJ : 2,08.
8. Avec les unités légales
Appareils
lampe
radiateur
Mini
perceuse
E : énergie (J)
7,2 × 104
2,25 × 106
1 920
P : puissance
(W)
40
2 500
16
t : durée (s)
1 800
900
120
E (Wh)
60
7. Lire avec un joulemètre-wattmètre
Valeurs
E 0,52
=
P 0,13
0
t (h)
0
1
2
3
4
5
b. Les grandeurs E et t sont proportionnelles, car la
droite représentative de E = f(t) passe par l’origine.
c. Le coefficient permettant de calculer E (Wh)
lorsqu’on connaît la durée t (h) est la puissance P de
l’ordinateur. Ici, P = 12 W.
d. L’énergie électrique transférée par l’ordinateur
pour un fonctionnement mensuel de 60 h est :
E = P.t = 12 × 60 = 720 Wh,
soit 50 fois moins qu’un ordinateur en forme de tour.
14. Lampadaire halogène (développement
durable)
a. La puissance transférée par des lampes LBC pour
9. Avec les unités pratiques
Appareils
Réacteur
nucléaire
Calculatrice
Lave-linge
E : énergie
216 GWh
10,8 J
3 kWh
P : puissance
900 MW
0,06 W
2 000 W
t : durée
10 jours
3 minutes
1,5 heure
obtenir la même quantité de lumière est de 60 W.
b. Il faudra 3 lampes LBC de 20 W pour remplacer le
lampadaire halogène.
Utiliser ses connaissances
15. Maîtriser l’énergie (développement
10. Coût d’un oubli
a. Énergie perdue : E = 30 × 24 × 0,1 = 72 kWh.
b. Le coût est de : 0,13 × 72 = 9,36 €.
11. Deux cycles d’un lave-vaisselle
a. La puissance électrique moyenne consommée
lors du cycle de lavage est :
700
P = 
= 1 400 W.
0,5
b. L’énergie électrique consommée par le lave-vaisselle lors du cycle de lavage est :
875 × 2 = 1 750 Wh ou 1,75 kWh.
c. Le coût du cycle rapide est de : 0,70 × 0,13 = 0,09 €.
Celui du cycle intensif est de : 1,75 × 0,13 = 0,23 €.
La différence de coût entre ces deux cycles est de
0,14 €.
durable)
a. Cet appareil est de la classe A.
b. Les machines les plus « énergivores » sont de la
classe G.
c. Durant un cycle de fonctionnement, une énergie
de 1,4 kWh est consommée.
d. Coût d’un cycle de lavage : 1,4 × 0,13 = 0,18 €.
16. La petite boulangerie
a. Dans la première pièce, la puissance transférée
est
P1 = 3 000 + 1 000 + 4 × 75 = 4 300 W.
Le produit : U.I = 230 × 22 = 5 050 W.
La puissance transférée est inférieure à U.I, car les
machines fonctionnent avec des moteurs donc la formule P = U.I n’est pas applicable et l’on a P < U.I.
18 • CHAPITRE 4 - Comment évaluer sa consommation d’énergie électrique ?
b. La puissance consommée dans la deuxième pièce
lorsque tous les appareils fonctionnent est
P2 = 2 400 + 800 + 5 × 60 = 3 500 W.
c. Le produit : U. × I = 230 × 15,3 = 3 500 W.
On a P = U.I : les appareils sont des dipôles résistifs.
Lorsque tous les appareils fonctionnent, la puissance
transférée est :
P1 + P2 = 4 300 + 3 500 = 7 800 W.
La puissance souscrite est de 9 000 W, donc ils peuvent fonctionner simultanément.
17. Petit déjeuner
a. Énergie consommée : E = P.t = 800 × 5 × 60
= 2,4 × 105 J.
P 800
= 3,48 A.
b. Intensité efficace : I =  =
U 230
U 230
= 66 Ω .
c. Résistance du grille pain : R =  =
I 3, 48
18. Veille voleuse (développement durable)
a. L’énergie consommée par jour :
- lorsque J. Blackwatt regarde la télévision :
E = P.t = 0,1 × 3 = 0,3 kWh ;
- lorsque le poste est en mode veille :
E = P.t = 0,016 × 21 = 0,336 kWh.
b. S’il éteint le poste, il fera une économie de :
0,336 × 0,13 = 0,04 € par jour, soit 16 € par an.
c. Autres appareils conçus sur le même type d’arrêt :
un décodeur, un ordinateur, un lecteur de DVD, une
box…
19. Consommation électrique d’une maison
(développement durable)
a. Les appareils consommant près du tiers de l’énergie électrique sont le réfrigérateur et le congélateur.
b. Diagramme circulaire à secteur représentant les
consommations électriques de la maison :
autres
lave-linge
7%
audiovisuel
7%
froid
32 %
12 %
- veiller à éloigner des sources de chaleur les appareils créant du froid (réfrigérateur, congélateur) ;
- éteindre chaque fois que cela est possible les
appareils en mode veille ;
- utiliser des lampes basse consommation (fluorescentes ou électroluminescentes) ;
- au moment du remplacement des appareils, choisir des appareils économisant l’énergie (classe A ;
A+) ;
- isoler thermiquement les habitations (voir la
réglementation technique en vigueur).
d. ADEME signifie : Agence De l’Environnement et
de la Maîtrise de l’Energie. Cette agence s’occupe des
déchets, de la pollution, de l’énergie, des matériaux
renouvelables, du développement durable…
20. Sur le Net ou au CDI
Se renseigner sur :
a. James Watt (1736 – 1819) a permis, par ses travaux,
l’amélioration de la machine à vapeur qui servait à
l’extraction du charbon dans les mines. Le nom de
Watt a été donné à l’unité de puissance.
James Joule (1818 – 1889), après des études scientifiques à l’Université de Cambridge, reprit l’entreprise
familiale de brasseur de bière et y construisit un laboratoire de recherche. Ses nombreuses études ont porté
sur l’énergie, la chaleur et l’électricité. Il a montré, en
particulier, que la chaleur est une forme d’énergie. Le
nom de Joule a été donné à l’unité d’énergie.
b. Thomas Alva Edison (1847 – 1931) est reconnu
comme l’un des inventeurs américains les plus
importants. C’est le fondateur de l’un des plus grands
empires industriels mondiaux : Général Electric. Il
revendique un nombre record de 1 093 brevets et
inventions : le télégraphe ; « transmetteur-receveur
duplex automatique de code Morse » ; le phonographe ; l’ampoule électrique ; le kinétographe ; le kinétoscope Parlors ; l’accumulateur alcalin nickel-fer …
21. Un atelier en hiver
sèche-linge
14 %
lave-vaisselle
14 %
14 %
éclairage
c. Précautions à prendre pour qu’une famille réalise
des économies d’énergie électrique :
- utiliser correctement les thermostats des appareils pour réguler la température ;
- utiliser des doubles ou triples vitrages ;
- L’appareil électrique de plus grande puissance est
la scie à bande avec 6 kW (Sylvestre Bobois ne peut
utiliser qu’une machine à la fois puisqu’il travaille
seul. De plus, il doit économiser l’énergie en éteignant les machines lorsqu’il ne les utilise pas).
- Le chauffage et l’éclairage consomment une puissance de 4 + 0,2 = 4,2 kW.
- Il utilise donc au maximum 4,2 + 6 = 10,2 kW. Avec
un abonnement de 12 kW, il peut donc s’éclairer et se
chauffer tout en travaillant en hiver.
CHAPITRE 4 - Comment évaluer sa consommation d’énergie électrique ? • 19
22. Vérifier sa facture avec un tableur (TIC)
1
2
3
4
5
6
7
8
9
10
11
12
B
C
D
E
F
G
H
I
J
ancien relevé nouveau différence consommation (kWh) prix kWh (�) montant HT (�) taxes locales TVA total TTC (�)
47,87
4,59
6,16
58,62
abonnement 4.45 � / mois
26,70
consommation du 26/10/08 au 25/04/09
10949
11218
269
269
0,0787
21,17
électricité
A
autres prestations
269
montant HT (�) taxes locales TVA
1,21
0,24
0,0045
1,21
TTC (�)
1,45
montant HT (�) taxes locales TVA
49,08
4,59
6,40
TTC (�)
60,07
total
23. Geste recyclage
a. Les lampes à recycler dans la filière lampes usagées sont les lampes fluocompactes, les tubes fluorescents et les lampes à DEL (sur leurs emballages
figure une poubelle barrée).
b. Les lampes fluocompactes et les tubes fluorescents contiennent du mercure. Celui-ci ne doit pas
se retrouver dans la chaîne alimentaire à cause de sa
haute toxicité.
c. Il faut déposer ces lampes dans un point de collecte prévu à cet effet dans les déchetteries ou les
magasins qui les vendent. Leur transfert et leur recyclage sont financés par l’éco-contribution payée lors
de l’achat.
trons avec des atomes de mercure produisent des
ultraviolets transformés en lumière visible en traversant une poudre fluorescente qui recouvre les parois.
p o udre
f l uo r e sce nt e
atome de mercure
gaz neutre
(argon)
p o udre
f l uo r e sce nt e
l um iè re b l a nch e
v isib l e
24. Rechercher
a. Le rapport des puissances consommées entre une
lampe à incandescence et une lampe basse consommation est de 5.
b. En 1802, Humphry Davy découvre que le passage
du courant électrique dans un fil conducteur élève
sa température et le fait fondre rapidement. En 1880,
Thomas Alva Edison invente la lampe à incandescence en plaçant une fibre de bambou du Japon dans
une ampoule sous vide. Cette fibre soumise à une
faible tension produit de la lumière. Ce filament fut
remplacé par la suite par un filament de tungstène.
Le passage du courant électrique produit un échauffement portant à incandescence le filament. La durée
de vie de ce type de lampe est de 1 000 à 2 000 heures.
c. Dans les lampes fluorescentes (aussi appelées
tubes ou lampes fluocompactes), des décharges électriques fournissent un courant. Les chocs des élec-
Les lampes électroluminescentes (DEL) : la jonction entre deux cristaux semi-conducteurs émet un
rayonnement lumineux lorsqu’elle est parcourue par
un courant. Enfermées dans une résine, ces lampes
résistent aux vibrations. Associées en nombre et commandées par l’électronique, les DEL sont devenues à
haute luminosité.
d. Les rendus des couleurs sont différents pour les
trois lampes. Le meilleur (proche de la lumière blanche naturelle) est celui de la lampe à incandescence.
Les spécialistes utilisent l’indice de rendu des couleurs (IRC) pour caractériser la lumière des lampes. Sa
valeur maximale 100 correspond à la lumière du jour.
Une lampe à incandescence ordinaire à un indice
supérieur à 95 alors que l’éclairage fluorescent va de
50 à 80 suivant la composition de la poudre contenue
dans la lampe. Les DEL ont un indice médiocre.
20 • CHAPITRE 4 - Comment évaluer sa consommation d’énergie électrique ?
5 Tous les sons sont-ils audibles ? CME3
Les activités
■■Activité 1 Comment déterminer la fréquence d’un son ?
Quelle est la gamme de fréquences d’un son audible
par l’homme ?
•Matériel
Un GBF
Un haut-parleur
Un interrupteur
Un microphone
Un oscilloscope
•Réponses aux questions
Expérience 1
1. Lorsque la fréquence augmente, le son devient plus aigu.
2. Oui, les deux fréquences sont égales.
Expérience 2
3. L’intervalle des fréquences des sons audibles est en théorie de [20 Hz ; 20 kHz]. En général, il est
plus restreint dans la réalité : une personne âgée n’entend plus les sons au-delà de 10 à 12 kHz.
4. Notre oreille n’est sensible qu’aux sons de certaines fréquences. En dessous de 20 Hz et au dessus
de 20 kHz, nous ne percevons pas les vibrations contrairement à certains animaux.
■■Activité 2 Quelle grandeur se mesure en décibel ?
•Matériel
Un GBF
Un haut-parleur
Un sonomètre
Un voltmètre
Une boîte d’expérimentation (si possible), dans laquelle on peut fixer un haut-parleur et possédant un
trou pour le capteur du sonomètre.
•Réponse aux questions
1. Non, le niveau d’intensité acoustique du « silence » n’est pas égal à zéro.
2. Lorsque la tension aux bornes d’un haut-parleur augmente, le niveau d’intensité acoustique
augmente. Lorsque le son est « plus fort », le niveau d’intensité acoustique augmente.
3. Nous ne percevons que certains sons, car notre oreille n’est sensible qu’à partir d’une valeur
minimale de niveau d’intensité sonore (voir le diagramme de Fletcher et Munson).
CHAPITRE 5 - Tous les sons sont-ils audibles ? • 21
Les exercices
Tester ses connaissances
Q.C.M
1 : C ; 5 : C ; 2 : A ; 6 : C ; 3 : A et B ;
7 : A, B, C.
4 : A ; Tester ses capacités
1. Un son pur
Le son pur correspond à l’oscillogramme a.
Le son complexe correspond à l’oscillogramme b.
Le bruit correspond à l’oscillogramme c.
2. Mesure d’une période sur un oscilloscope
Les enregistrements a et b sont périodiques. Pour
chacun de ces enregistrements, un motif est dessiné
ci-dessous :
a. b.
est la plus basse. Le classement des sons du plus
grave au plus aigu est donc : son 4 ; son 2 ; son 5 ;
son 3 ; son1.
7. Sans calcul
Les sons les plus graves sont ceux dont la période est
la plus grande. Le classement des sons du plus aigu
au plus grave est donc : d ; b ; a ; c.
8. Un seul mot
a. Pour augmenter la hauteur d’un son, il faut augmenter sa fréquence. Pour diminuer la hauteur d’un
son, il faut diminuer sa fréquence.
b. Un son aigu a une fréquence comprise entre
1,5 kHz et 20 kHz.
c. Les ultrasons ont une fréquence supérieure à
20 kHz.
d. En dessous de 20 Hz, un son n’est pas audible par
l’homme.
9. Calcul de fréquences
a. Enregistrement a : Ta = 2 ms ; enregistrement b :
Tb = 2 ms ; enregistrement c : Tc = 20 ms ;
b. Enregistrement a : fa = 500 Hz ; enregistrement b :
Appliquer le cours
fb = 500 Hz ; enregistrement c : fc = 50 Hz.
3. Mesure d’une période à partir d’un
c. Les sons a et b sont plus aigus que le son c.
d. La période diminue quand le son devient plus
enregistrement EXAO
a. Pour l’enregistrement 1, T = 2,23 ms d’où
aigu.
f = 448 Hz.
b. Pour l’enregistrement 2, T = 2,25 ms d’où
f = 444 Hz.
10. Fletcher et Munson
a. Pas toujours. Par exemple, un son de 50 Hz et
4. Comment positionner un sonomètre ?
a. La bonne position est celle schématisée sur le dessin 3.
b. Il faut paramétrer le sonomètre pour qu’il mesure
en dB(A).
5. Quels réglages pour quels effets ?
a.
HP
GBF
b. Il faut régler la fréquence de la tension de sortie du
GBF.
c. Il faut régler la valeur de la tension de sortie du
GBF (bouton « Amplitude »).
6. Mettre de l’ordre
Les sons les plus graves sont ceux dont la fréquence
22 • CHAPITRE 5 - Tous les sons sont-ils audibles ?
40 dB n’est pas audible.
b. Non. Seulement si L > 5 dB.
c. Par exemple, un son de (3 000 Hz ; 95 dB).
d. Entre 2500 et 4000 Hz.
11. Quel bouton ?
Les réglages sont ceux de la photo.
a. C’est une tension alternative sinusoïdale (bouton
rouge de droite).
b. La fréquence est donnée à l’écran. Ici f = 400 Hz.
c. Le haut-parleur émet un son de fréquence égale
à celle de la tension qui l’alimente. Donc ici, la fréquence du son émis vaut 400 Hz.
Les réglages sont modifiés :
d. Non, cela va augmenter ou diminuer le niveau
d’intensité sonore, mais pas la fréquence.
e. Il faut se placer sur le calibre 100 kHz et tourner
le bouton fréquence jusqu’à ce que l’écran affiche
14 000.
f. fmax = 100 kHz. Dans ce cas, le son est inaudible,
car f > 20 kHz. C’est un ultrason.
g. fmin = 1 Hz. Le son est un infrason.
12. Additivité de deux sources sonores
b. Un son aigu a une fréquence comprise entre
a. « Si deux sources sonores émettent des sons de
1,5 kHz et 20 kHz. Quentin doit donc positionner les
curseurs 3 kHz et 10 kHz au maximum.
même niveau d’intensité sonore, le niveau d’intensité sonore total augmente de trois. »
b. « Si deux sources sonores émettent des sons dont
le niveau d’intensité sonore diffère de 10 dB, le niveau
d’intensité sonore global est sensiblement égal à la
valeur du niveau d’intensité sonore de la source la
plus forte. »
16. Arthur accorde sa guitare
a. La période de ce signal vaut T = 3 ms. La fréquence
est donc égale à :
1
1
=
= 333 Hz.
T 3 × 10 −3
b. Il faut qu’Arthur diminue la fréquence du son
émis par la corde. Pour cela, il doit la détendre afin
d’allonger la partie vibrante de la corde.
f=
13. Quel aspirateur ?
a. Daison, car c’est pour celui là que le niveau d’intensité acoustique L est le plus petit (74 dB).
17. Jouer de la guitare sèche
b. Non. Ces deux grandeurs ne sont pas liées.
a. Oui, car les sons médiums ont une fréquence com-
14. Un haut-parleur adapté
prise entre 300 Hz et 1 500 Hz. En plaçant ses doigts
sur la corde, Arthur va la raccourcir et donc augmenter la fréquence du son émis.
b. Non, car la fréquence du son ne peut qu’augmenter si on raccourcit la partie vibrante de la corde. Or les
sons graves ont une fréquence inférieure à 300 Hz.
a. Non, car un son grave a une fréquence comprise
entre 20 Hz et 300 Hz.
b. Non, car les fréquences d’une conversation sont
comprises entre 250 Hz et 3 500 Hz. Ce haut-parleur
ne retransmettra que les sons les plus aigus de la
conversation.
c. Non, car les ultrasons ont des fréquences supérieures à 20 000 Hz.
d. Oui, car les douleurs apparaissent environ à 120 dB.
18. Situation problème
a. Oui, car la détérioration des capacités auditives
commence à environ 80 dB.
b. Le responsable doit fournir des dispositifs de protection individuelle contre le bruit et s’assurer que les
employés les utilisent.
15. Régler la hauteur du son d’un téléviseur
a. La fréquence d’un son grave est comprise entre
20 Hz et 300 Hz. Quentin doit donc positionner le
curseur 100 Hz au maximum.
6 Comment isoler une pièce du bruit ? CME3
Les activités
■■Activité 1Le double vitrage assure-t-il une meilleure isolation phonique
que le simple vitrage ?
•Matériel
Un GBF - Un haut-parleur - Un interrupteur - Un sonomètre - Une caisse
Exemple de relevé des niveaux d’intensité acoustique (en décibel) pour différents vitrages
Fréquence (Hz)
125
500
1 000
4 000
8 000
Air
82,7
90
90
90
83
Air ou vitrage
Verre 4 mm
59,5
48,1
59
45,9
49,9
Verre 10 mm
53,5
43,2
55,6
41,1
45,2
Double vitrage 4/10/4
55,5
47,5
57,3
43,3
46,8
Double vitrage 10/10/4
50,7
40,2
53,1
39,2
41,2
CHAPITRE 6 - Comment isoler une pièce du bruit ? • 23
•Réponses aux questions
Expérience 1
1. Oui, le verre est un isolant acoustique : L(verre) < L(air).
2. Oui, l’épaisseur du verre joue sur le niveau d’isolement : L(verre 10 mm) < L(verre 4 mm).
3. Non, chaque matériau a une fréquence de résonance pour laquelle il isole moins bien.
Expérience 2
4. Non, un double vitrage 4/10/4 n’isole pas mieux qu’un simple vitrage de 10 mm.
■■Activité 2Comment évolue le niveau d’intensité acoustique lorsque
le récepteur s’éloigne de la source sonore ?
•Matériel
Un GBF
Un haut-parleur
Un interrupteur
Un sonomètre
Une caisse
Distance d entre le haut-parleur et le sonomètre (cm)
Niveau sonore L en dB (arrondi à l’unité)
5
80
10
74
20
68
40
62
•Réponses aux questions
1.
L (dB)
80
75
70
65
60
d (cm)
0
0
5
10
20
40
2. Le niveau d’intensité acoustique diminue lors que la distance entre la source et le récepteur
augmente. Cette diminution est de 6 décibels chaque fois que la distance double.
24 • CHAPITRE 6 - Comment isoler une pièce du bruit ?
6. Épaisseur d’air (les indices)
Les exercices
Tester ses connaissances
a. Dans ce cas Rw ≈ 41,5 dB.
b. Rw augmente d’environ 3,75 dB chaque fois que
Q.C.M.
1 : C ; 2 : B ; 3 : A et C ; 4 : B ; 5 : B.
7. Isolation par parois doubles
l’épaisseur du verre double.
a. Quand on remplace la couche d’air par un maté-
Tester ses capacités
riau isolant, on augmente l’indice d’affaiblissement
acoustique.
1. Le meilleur matériau
Le classement des matériaux du plus performant au
moins performant est : matériau 3 ; matériau 4 ; matériau 1 ; matériau 2.
2. L en fonction de d
a. Le niveau d’intensité acoustique L diminue lorsque la distance d entre la source sonore et le récepteur augmente.
b.
Distance de mesure
d (m)
2
4
16
32
Niveau d’intensité
acoustique L (dB)
98
92
80
74
3. Vrai – Faux
Le niveau d’intensité acoustique :
a. Faux : se note L.
b. Faux : se mesure avec un sonomètre.
c. Faux : diminue lorsque la distance augmente.
d. Vrai : décroît lorsque la distance augmente.
e. Faux : augmente lorsque la distance diminue.
Appliquer le cours
4. Dans un immeuble
a. Il doit intervenir pour que l’isolement phonique
entre deux appartements soit au moins de 53 dB et
que l’isolement entre la cage d’escalier et un appartement soit au moins de 53 dB (voir le document sur
les niveaux règlementaires d’isolation, page 72 du
manuel) :
- entre le studio 1 et le studio 2 ;
- entre le studio 2 et la cage d’escalier ;
- entre le studio 4 et la cage d’escalier.
b. Nicolas devrait faire mesurer
- le niveau d’isolement des façades ;
- le niveau d’isolement des sols et des plafonds aux
bruits d’impacts et aux bruits aériens.
5. Épaisseur du vitrage
a. Le verre isole plus des sons aigus que des sons graves, car les sons aigus ont des fréquences plus élevées
que les sons graves.
b. Plus le verre est épais, plus l’atténuation phonique
est importante.
c. L’atténuation dans ce cas est de 40 décibels.
b. Non les deux schémas centraux montrent que
deux murs identiques en série n’isolent pas deux fois
plus qu’un mur seul.
c. Les deux plaques de plâtre intérieures sont reliées
aux plaques de plâtre extérieures par les poteaux.
Lorsque le son percute la paroi, l’onde sonore est
transmise à la plaque de plâtre intérieure par le
poteau. Elle traverse donc la cloison. Si la plaque de
plâtre intérieure est enlevée, l’onde sonore n’est plus
transmise par ce biais et l’atténuation sonore augmente.
8. Loi des masses
a. Les trois paramètres qui influent sur l’isolation
acoustique d’une paroi sont : la fréquence du son,
l’épaisseur de la paroi, la qualité du matériau utilisé.
b. L’atténuation est meilleure pour les sons aigus,
d’après le diagramme.
c. Dans ce cas, l’affaiblissement Rw a une valeur de
28 dB.
d. On peut utiliser de la brique pleine de 5,5 centi­
mètres d’épaisseur.
9. Calfeutrage
a. Faux. Le son n’a besoin que de petits espaces pour
se propager : « Là où l’air passe, le bruit passe ».
b. Faux. Si l’air passe sous une porte, le bruit passe
forcément.
c. Vrai. Des fenêtres étanches à l’air sont indispensables pour améliorer l’isolation phonique.
d. Vrai. Le calfeutrage parfait d’une pièce est bon
pour l’isolation mais mauvais pour la santé. Une
pièce saine doit être aérée.
10. Capotage insonorisant
a. Les principes évoqués sont :
- la loi de masse : plus le poids au m² d’une cloison
est important, plus elle sera efficace pour isoler des
sons ;
- la loi d’étanchéité : une paroi doit être parfaitement étanche pour isoler du bruit.
b. Le plomb est le meilleur isolant.
c. Oui, une petite fuite peut limiter grandement
l’isolement du capot.
CHAPITRE 6 - Comment isoler une pièce du bruit ? • 25
Utiliser ses connaissances
11. Connaître et respecter les limites
a. dB signifie décibel. C’est l’unité du niveau d’intensité sonore L.
b. Non, le niveau d’intensité sonore au calme à la
campagne est d’environ 30 dB.
c. Le seuil de fatigue auditive est de 85 dB.
d. Dans un bureau : L ≤ 40 dB.
e. Dans un atelier d’ajustage : 8 h maximum. Dans
une chaudronnerie : 2 minutes maximum.
f. Oui. (L = 105 dB.)
g. Test de moteur sur un banc d’essai.
h. Écouter un baladeur, aller en discothèque, suivre
un concert de rock, chasser.
12. Rénover son appartement
a. En 2 : permuter RW et LW : Rw est un indice d’affaiblissement des bruits aériens et Lw un indice d’affaiblissement aux bruits d’impacts. Il ne faut pas
confondre les indices de performance des matériaux
avec les performances de la paroi.
b. Afin d’assurer l’étanchéité des ouvertures et limiter le passage de l’onde sonore.
c. Il pourrait isoler les planchers et les plafonds,
remplacer les vitrages par des vitrages spécifiques.
13. Trafic aérien.
a. À 300 mètres, L ≈ 103 dB.
b. À 18 m, L18 ≈ 127 dB. À 1 200 m, L1200 ≈ 91 dB.
À 4,8 km, L4800 = 79 dB.
Méthode de calcul destinée au professeur
L’intensité acoustique I est inversement proportionnelle au carré de la distance d :
I = k/d2 où k est une constante.
Le niveau d’intensité acoustique est donné par :
L = 10 log(I/I0) avec I0 = 10–12 W.m–2.
k
k
L = 10 log 2
= 10 log (
) – 20 log d = K – 20 log d
I0
d . I 0
où K est une autre constante.
Pour la distance d1, L1 = K – 20 log d1.
Pour la distance d2, L2 = K – 20 log d2.
L2 – L1 = 20 log (d1/d2).
L’abaque donne L1 = 103 dB pour d1 = 300 m, lors du
décollage.
Calculons L2 pour d2 = 18 m :
L2 – L1 = 20 log (300/18) = 20 log 16,7 = 24.
Donc L2 = 127 dB.
Calculons L3 pour d3 = 1 200 m :
L3 – L1 = 20 log (300/1 200) = –20 log 4
= –20 log 22 = –40 log 2 = –40 × 0,3 = –12.
Donc L3 = 91 dB.
Calculons L4 pour d4 = 4 800 m :
L4 – L1 = 20 log (300/4 800) = –20 log 16 = –20 log 24
= –80 log 2 = –24.
26 • CHAPITRE 6 - Comment isoler une pièce du bruit ?
Donc L4 = 79 dB.
Méthode de calcul adaptée à l’élève
On procède par itération sachant que le niveau diminue de 6 dB quand la distance double.
- Évaluation du niveau d’intensité acoustique L2 à
18 m, connaissant le niveau L1 = 103 dB à 300 m :
Distance (en m)
Niveau d’intensité acoustique (dB)
18
L2
36
L2 – 6
72
L2 – 12
144
L2 – 18
288
L2 – 24
La distance 288 m est voisine de 300 m et le niveau
d’intensité acoustique est voisin de L1 = 103 dB.
Donc L2 – 24 ≈ 103, soit L2 ≈ 127 dB.
NB : Ce résultat constitue une bonne approximation,
car la fonction logarithmique décroît lentement avec
la distance.
- Évaluation du niveau d’intensité acoustique L3 à
1200 m, connaissant le niveau L1 = 103 dB à 300 m :
Distance (en m)
Niveau d’intensité acoustique (dB)
300
L1
600
L1 – 6
1 200
L1 – 12
L3 = L1 – 12 = 103 – 12 = 91 dB.
- Évaluation du niveau d’intensité acoustique L4 à
4 800 m, connaissant le niveau L1 = 103 dB à 300 m :
Distance (en m)
Niveau d’intensité acoustique (dB)
300
L1
600
L1 – 6
1 200
L1 – 12
2 400
L1 – 18
4 800
L1 – 24
L4 = L1 – 24 = 103 – 24 = 79 dB.
c. Isolement = 79 – 35 = 44 dB. L’isolement d’une
chambre située à 4,8 kilomètres doit être au moins
égal à 44 décibels.
14. Cloisons d’isolation phonique
a. La cloison F530 offre les meilleures performances
acoustiques (courbe rouge).
b. L’indice de la cloison 72/48 chute pour les fréquences supérieures à 2000 Hz.
c. Oui, car une partie seulement de la bande de fréquences des sons des conversations sera mal atténuée.
d. L’isolation permet une isolation de 40 à 70 décibels selon les fréquences. Cette cloison permet donc
une isolation convenable.
e. Non, car il ne faut pas confondre l’indice d’isolement des matériaux et l’isolement du mur.
7 Comment éviter le basculement
d’un corps ? HS1
Les activités
■■Activité 1 À quelle condition un objet peut-il basculer ?
•Matériel
Un parallélogramme déformable muni d’un fil à plomb
Expérience
- Présenter le parallélogramme déformable, son centre de gravité G ainsi que la verticale de G
matérialisée par le fil à plomb. Présenter la surface d’appui sur le plan, appelée base de sustentation.
- Déformer progressivement le parallélogramme jusqu’à ce qu’il se renverse.
•Réponses aux questions
1. Le parallélogramme reste en équilibre sur le plan horizontal tant que la verticale du centre de
gravité, matérialisée par le fil à plomb, rencontre la base de sustentation.
2. Le parallélogramme bascule dès que la verticale du centre de gravité, tombe en dehors de la base
de sustentation.
3. Lorsque le camion a déployé la grue, le centre de gravité du système camion-grue s’est déplacé
et sa verticale est sortie de la base de sustentation comprise entre les quatre roues. Le camion s’est
alors renversé. Le camionneur aurait dû sortir les stabilisateurs pour augmenter l’aire de la base de
sustentation.
■■Activité 2 Quelles sont les caractéristiques du poids d’un corps ?
•Matériel
Un support, une noix, une tige
Un fil à plomb
Une boulette de pâte à modeler
Une boîte d’allumettes
Un dynamomètre
Une balance électronique
Expérience
- Matérialiser la direction du fil à plomb par un petit morceau de pâte à modeler. Remonter le fil et le
brûler. Vérifier que le point d’impact coïncide avec la trace marquée sur la pâte à modeler.
- Mesurer le poids du fil à plomb avec un dynamomètre.
- Mesurer sa masse avec une balance électronique.
•Répondre aux questions
Expérience 1
1. Le poids du fil à plomb s’applique au centre de gravité.
2. Le fil à plomb tombe suivant la direction verticale.
3. Le sens de la chute est du haut vers le bas.
4. La valeur du poids du fil à plomb est P = 1,6 N.
Expérience 2
5. La valeur de la masse du fil à plomb est m = 0,159 kg.
6. La valeur du rapport
P
1,6
= 
= 10 N/kg.
m
0,159
CHAPITRE 7 - Comment éviter le basculement d’un corps ? • 27
Les documents
■■Les plaques de charge sur les chariots frontaux
•Répondre aux questions
1. La hauteur de gerbage est de 3,20 m, inférieure à 3,30 m. C’est donc la courbe 1 qu’il faut utiliser.
Le centre de gravité G est situé à 600 mm du mât.
L’ordonnée correspondant à l’abscisse 600 mm sur la courbe 1 est 1 800 kg.
La charge maximale mesurant 1,2 m de côté pouvant être gerbée à 3,20 m est 1 800 kg.
2. La hauteur de gerbage est maintenant de 5,20 m, inférieure à 5,44 m. C’est donc la courbe 2 qu’il
faut utiliser. Le centre de gravité G est situé à 700 mm du mât. L’ordonnée correspondant à l’abscisse
700 mm sur la courbe 2 est 1 400 kg ; c’est la charge maximale que l’on peut gerber.
Par conséquent, il est impossible de gerber la charge de 1 800 kg à une hauteur de 5,20 m.
■■Deux risques liés à l’utilisation des grues mobiles
•Répondre aux questions
1. Une flèche de grue trop longue déplace le centre de gravité du système en dehors de la base de
sustentation.
2. Le conducteur doit sortir les quatre poutres des stabilisateurs pour augmenter l’aire de la base de
sustentation.
Les exercices
Tester ses connaissances
6. Recopier et compléter
Q.C.M.
1. : B ;
5. : B et C ;
Le poids d’un corps est représenté par un segment

fléché P dont l’origine correspond au centre de gravité, la direction et le sens sont ceux du poids et la
longueur est proportionnelle à la valeur du poids.
2. : C ;
6. : A et B.
3 : C ;
4. : B ;
Tester ses capacités
7. Représenter le poids d’une boule de
1. Centre de gravité d’un cylindre
Le segment fléché a pour origine le
point G, pour direction la verticale
de G et de sens vers le bas.
Le centre de gravité est situé au milieu du segment
qui joint les centres des deux bases du cylindre.
2. Centre de gravité d’un anneau
a. Le centre de gravité est situé au centre de l’anneau.
b. Le centre de gravité ne se trouve pas dans la
matière de l’anneau.
3. Centre de gravité d’une plaque triangulaire
Il faut tracer les médianes du triangle pour déterminer son centre de gravité.
4. Choisir la bonne réponse
Le poids d’un corps se mesure avec un dynamomètre.
La valeur du poids d’un corps s’exprime en newton.
5. Lire un appareil de mesure
Le poids de l’objet suspendu en (a) est 4 N.
Le poids de l’objet suspendu en (b) est 2,05 N.
pétanque.
G
Avec l’échelle choisie le segment
fléché a pour longueur :
0,7
= 3,5 cm.
0,2
8. Stabilité d’un équilibre
Le guéridon est moins stable que la
table basse, car son centre de gravité est plus haut et sa base de sustentation plus réduite.
P
9. Basculement frontal
La charge à déplacer a été mal positionnée : la verticale du centre de gravité du système tombe en avant
de la base de sustentation formée par la zone intérieure aux quatre roues. Le chariot bascule autour
des roues avant.
28 • CHAPITRE 7 - Comment éviter le basculement d’un corps ?
Appliquer le cours
b. Si l’échelle n’est pas suffisamment inclinée, la
10. Mesure du poids
a. L’appareil de mesure photographié est un dynamomètre.
b. Il faut multiplier la lecture par le facteur 0,1 pour
obtenir le poids.
c. L’objet suspendu pèse : 3,5 × 0,1 = 0,35 N.
11. Calcul du poids à partir de la masse
a. Dans la relation P = m.g, P représente la valeur du
poids de l’objet, m, la valeur de la masse et g, celle de
l’intensité de la pesanteur.
b. P s’exprime en newton, m en kilogramme et g en
newton par kilogramme.
c. La valeur du poids de l’objet est
P = 0,75 × 9,8 = 7,35 N.
12. Représentation du poids d’une charge
base de sustentation est réduite. Lors de travaux de
perçage, la personne exerce une action contre le mur.
En réaction, elle a tendance à s’éloigner du mur et la
verticale du centre de gravité peut facilement sortir
de la base de sustentation ce qui entraîne le basculement de l’échelle.
15. Équilibre d’un objet lesté
Dans la position 1 :
a. la base de sustentation est le point O ;
b. la verticale du point G rencontre le point O ;
c. l’objet reste en équilibre.
Dans la position 2 :
d. la base de sustentation est le segment AB ;
e. la verticale du centre de gravité ne rencontre pas
la base de sustentation ;
f. l’objet ne peut pas rester en équilibre, il se redresse.
a. Le bloc de béton a une forme rectangulaire. Il possède deux axes de symétrie. Lorsqu’un solide homogène présente un axe de symétrie, son centre de gravité
se trouve sur cet axe. Le centre de gravité se trouve
donc à l’intersection des deux axes de symétrie. Il coïncide aussi avec les deux diagonales du rectangle.
b. P = m . g = 500 × 10 = 5 000 N.
c.
Point
d’application

P
G
Droite
d’action
Verticale
Sens
Valeur
De haut
en bas
5 000 N
A
Utiliser ses connaissances
16. Inclinaison de l’échelle
a. Voir ci-contre (échelle : 50%).
A
B
Graphiquement on trouve :
 = 71°.
BHM

ABH = BHM
b. 
.
AH
7
=
= 2, 8 .
AB 2, 5

ABH = 70,3°.
tan 
ABH =
17. Stabilisateurs
a. Le chef de chantier reproche au
H
M
grutier de ne pas avoir sorti les deux stabilisateurs de
la grue.
b. Cette négligence peut avoir pour conséquence un
renversement de la grue.
c. Sur la photo de la page 83, on observe que le grutier
n’a pas sorti les stabilisateurs. La base de sustentation
était trop réduite et lorsque la grue a été déployée, la
verticale du centre de gravité est sortie de la base de
sustentation et la grue s’est renversée.
G
P
13. Poids d’un panneau
solaire
a. Le poids du panneau est :
18. Le tracteur et l’épareuse
G
a. Lors du déploiement de l’épareuse, le centre de
P = m.g = 40 × 10 = 400 N.
b. Voir ci-contre (échelle : 50%).
P
14. Montée sur une échelle simple
inclinée
a. Lorsque la personne monte sur l’échelle, le centre
de gravité du système échelle-personne s’élève.
gravité du système tracteur-épareuse se déplace vers
l’épareuse.
b. Pour que le système ne se renverse pas, la verticale
du centre de gravité doit rester entre les quatre roues
du tracteur.
c. Le fait de gonfler les roues du tracteur à l’eau ou
de placer des masses à l’avant permet de diminuer le
déplacement du centre de gravité lorsqu’on déploie
l’épareuse.
CHAPITRE 7 - Comment éviter le basculement d’un corps ? • 29
19. Plaque de charge
20. Les grues mobiles
a. Pour une hauteur de 6,3 m c’est la courbe (2) qui
a. Au niveau de la grue, le centre de gravité doit res-
donne le résultat. Pour une distance de 600 mm, la
charge maximale est de 1 000 kg, soit 1 tonne.
b. Pour une hauteur de 4,9 m, c’est la courbe (1) qui
donne le résultat. Non le chariot ne peut pas élever
une charge de 1,2 tonne dont le centre de gravité est
situé à 0,7 m de la fourche, car le maximum est de
1 000 kg, soit une tonne.
ter entre les stabilisateurs.
b. La charge maximale que la grue peut soulever
pour une portée de 18 m est 1,7 tonne.
c. Si la grue doit soulever une masse de 20 tonnes, on
doit choisir une longueur de flèche de 8,6 m.
8 Comment soulever un objet ? HS1
Les activités
■■Activité 1Comment caractériser et représenter une action mécanique ?
•Matériel
Différents documents présentant des situations où s’exercent des actions mécaniques
Un aimant
Une bille d’acier
Un dynamomètre circulaire
Expérience 1
Présenter différents documents montrant des situations où s’exercent des actions mécaniques de
contact et à distance. Amener les élèves à préciser, qui exerce l’action et qui la subit, ainsi que les
effets qui en découlent.
On peut placer une bille d’acier sur une goulotte et la faire rouler près d’un aimant pour montrer
qu’il existe des actions mécaniques à distance.
Expérience 2
Analyser l’action mécanique exercée par la main sur le fil du dynamomètre, afin de préciser les quatre
caractéristiques d’une force.
•Réponses aux questions
Expérience 1
1. Dans le doc. 1, ce sont les câbles qui exercent l’action et le container qui la subit.
Les câbles sont au contact du container. L’effet provoqué est le déplacement du container.
Dans le doc. 2, c’est l’aimant qui exerce l’action et la bille qui la subit.
L’effet provoqué est une modification du mouvement. Cette action s’effectue à distance.
Dans le doc. 3, c’est la presse d’emboutissage qui exerce l’action et la tôle qui la subit. Cette action
provoque la déformation de la tôle.
Expérience 2
2. Le point d’application de la force est le point P ; sa droite d’action, la direction du fil ;
son sens, vers la droite et sa valeur 4 newtons.
3. Il faut déterminer les quatre caractéristiques d’une action mécanique : le point d’application,
la droite d’action, le sens et la valeur pour la représenter par une force.
30 • CHAPITRE 8 - Comment soulever un objet ?
■■Activité 2Quelles sont les conditions d’équilibre d’un objet
soumis à deux ou trois forces ?
•Matériel
Deux dynamomètres magnétiques avec leur fil muni d’un crochet
Une plaque de polystyrène
Une plaque triangulaire dont le centre de gravité est matérialisé
Un support magnétique avec un fil à plomb
Expérience 1
Rechercher les caractéristiques des deux forces s’exerçant sur la plaque, en particulier, leur direction,
leur sens et leur valeur.
Expérience 2
Maintenir la plaque triangulaire en équilibre à l’aide de deux dynamomètres. Positionner le fil à
plomb afin qu’il passe par le centre de gravité de la plaque. Il matérialise la direction du poids de la
plaque. Vérifier que les directions des deux forces et du poids sont concourantes.
•Réponse aux questions
Expérience 1
1. Les forces exercées en A et B ont même droite d’action, même valeur mais des sens opposés.
Expérience 2



2. Les droites d’action des trois forces P , F1 et F2 sont concourantes.
■■Remarque sur l’essentiel du cours
L’étude des vecteurs n’étant pas au programme de mathématiques de la classe de seconde
professionnelle, nous avons supprimé toutes les expressions vectorielles concernant :
- le principe des actions mutuelles ;
- l’équilibre d’un solide soumis à deux ou à trois forces.
Les exercices
Tester ses connaissances
Tester ses capacités
1. Le ressort
sort est une action de contact.
b. Elle est ponctuelle, car elle s’exerce à l’extrémité
du ressort.
2
a. L’action mécanique exercée par la main sur le res-
NEWTONS
Les caractéristiques de la force sont :
- le point d’application : le point P ;
- la droite d’action : l’horizontale ;
- le sens : vers la droite ;
- la valeur : 6 N.
6 : C.
8
5 : B ;
10
4 : C ;
6
3 : C ;
3. Le dynamomètre
4
Q.C.M.
1 : A ; 2 : B ;
P
Fmain/tige
(échelle : 50%)
2. Actions mécaniques
Action mécanique
Type d’action
Effets
1. Action de l’aimant sur la bille en acier Action à distance, répartie sur toute la
bille
Modification de la trajectoire de la bille
2. Action des câbles sur le container
Action de contact localisée
Mise en mouvement du container
3. Action de la presse d’emboutissage
sur la tôle.
Action de contact répartie
Déformation de la tôle
4. Action des haubans sur le tablier du
pont.
Action de contact localisée.
Maintien du tablier au repos
CHAPITRE 8 - Comment soulever un objet ? • 31
4. Représenter une force
10. Réaction du support
a. Les deux forces qui s’exercent sur la boule sont le
C
Fcorde/crochet
5. Le marteau
(échelle : 50 %)
O
poids de la boule et la réaction du plan.
b. Tableau des caractéristiques des forces :
Point
d’application
Droite
d’action
Sens
Valeur
Poids de
la boule
G
Verticale
De haut
en bas
3 N
Réaction
du plan
O
Verticale
De bas
en haut
3 N
c.
R
F marteau/clou
6. Prévoir un état d’équilibre
G
a. Pour qu’un solide soumis à deux forces soit en
équilibre, il faut que ces deux forces aient :
- même droite d’action ;
- même valeur ;
- des sens opposés.
b. Le seul solide en équilibre est le solide 2 qui présente toutes les propriétés requises. Dans le schéma 1
les forces n’ont pas même droite d’action ; dans le
schéma 3, les forces ont même droite d’action, des
sens opposés mais n’ont pas la même valeur ; dans le
schéma 4, les deux forces ont même sens.
7. Un état d’équilibre.
O
P
11. Tension du ressort
a. Les deux forces quis’exercent sur la boule sont le
poids P et la tension T du ressort.
b. Tableau des caractéristiques des forces :
a. Un solide soumis à deux forces est en équilibre si
ces deux forces ont :
- même droite d’action ;
- même valeur ;
- des sens opposés.
b. Le solide (S) est en équilibre, car les deux forces
ont des droites d’action identiques, même valeur :
5 N et des sens opposés.
Point
d’application
Droite
d’action
Sens
Valeur

P
G
Verticale
De haut en
bas
5 N

T
A
Verticale
De bas en
haut
5 N
T
8. Dynamique des forces
C’est le système (b) qui est en équilibre, car le dynamique des forces est fermé.
Appliquer le cours
9. Réaliser un inventaire des forces

F1  : force exercée par Alain sur la corde (C) notée


F1 =  FA/C .

F2  : force exercée par Bernard sur le sol (S) notée


F2 =  FB/S .

F3  : force exercée par Alain sur le sol (S) notée


F3 =  FA/S .

F4  : force exercée par Bernard sur la corde (C) notée


F4 =  FB/C .
32 • CHAPITRE 8 - Comment soulever un objet ?
A
G
P
12. Boules électrisées en interaction
a. Le principe des actions mutuelles dit que les deux


forces FA/B et FB/ A ont même droite d’action, même
valeur mais des sens opposés :
b.
c. Dynamique des forces :
d. Graphiquement la longueur des
Les valeurs sont égales à 2 N.


vecteurs F1 et F2 mesure 2,1 cm.
Leur valeur est donc :
200 × 2,1 = 420 N.
On peut alors reporter ces valeurs
dans le tableau des caractéristiques
des forces.
e. La législation n’est pas respectée,
car les livreurs doivent exercer une
force de 420 N, supérieure à celle prévue par la législation (350 N).
13. Force magnétique
15. Équilibre d’une échelle
a. La force TC /B exercée par le câble sur la bille est
a. b.
(A)
FB/A
(B)
F A/B


c. Les sens de FA/B et de FB/ A sont opposés.

appelée la tension du câble.
b. Tableau des caractéristiques des forces :
Droite
d’action
Sens
Valeur

P
G
Verticale
4 N

FA/B
De haut
en bas
G
Horizontale
2,3 N

TC /B
Vers la
gauche
P
c. Dynamique des forces :
d. Graphiquement, la
P
3
e. Cos 30° =  = 
d’où
T
2
3
T = 2P
= 4,62 N
3
F = ½ T = 2,31 N.
F1
C
x
Voir ci-contre (échelle : 50 %) :
B
G
4,6 N

égale 2,3 N et celle de TC /B
égale 4,6 N.
P
20°
Point
d’application

valeur de la force FA/B
F2
30°
c. Puisque l’échelle est en équi-
P
d. Tableau des caractéristiques des forces :
Point
d’application
Droite
d’action
Sens
Valeur

P
G
Verticale
1 000 N

FM /E
De haut
en bas
B
Horizontale
120 N

FS/E
De droite
à gauche
A
TC/B
P
FA/B
A
libre, les droites d’action des
 

trois forces P , FM /E et FS/E sont
concourantes. La droite d’action de

FS/E est donc AC.
1 000 N
e. Dynamique des forces :
FM/E
Utiliser ses connaissances
14. Livraison d’un colis
a. La valeur du poids de la gazinière est :
P = m . g = 80 × 10 = 800 N.
b. Tableau des caractéristiques des forces :
Point
d’application
Droite
d’action
Sens
Valeur

P
G
Verticale
800 N

F1
De haut en
bas
A
420 N

F2
B
420 N
Graphiquement on trouve :

FM /E mesure 0,6 cm,
soit 0,6 × 200 = 120 N ;

FS/E mesure 5 cm soit
5 × 200 = 1 000 N.
FS/E
P
On peut alors reporter ces valeurs
dans le tableau des caractéristiques
des forces.
CHAPITRE 8 - Comment soulever un objet ? • 33
16. Résistance d’un câble
F1/60° = 5,8 × 100 = 580 N.
On peut retrouver ces valeurs par le calcul :
3
F1/60° = 2 × 500 ×
= 577 N.
3


b. Valeurs des forces de F1 et F2 pour différentes
valeurs de a :
a. Tableau des caractéristiques des forces :
Point
d’application
Droite
d’action
Sens
Valeur
Verticale
De haut
en bas
4 600 N

P
G

F1
A
4 000 N

F2
A
2 300 N
b. Dynamique des forces
a

Valeur de P

Valeur de F1

Valeur de F2
F2
(échelle : 50%) :
30°
45°
60°
1 000 N
1 000 N
1 000 N
1 000 N
700 N
580 N
1 000 N
700 N
580 N
c. Les valeurs des forces sont les plus importantes
c. Graphiquement
la longueur

de P mesure 9,2 cm.
La valeur maximale du poids
que l’on peut suspendre est
donc :
9,2 × 500 = 4 600 N.
On peut retrouver cette valeur
par le calcul dans le demitriangle équilatéral PF1F2 :
P
F1
pour un angle de 30°. Ce résultat est général : les
efforts à supporter par les élingues sont d’autant plus
importants que l’angle entre l’élingue et la pièce à
soulever est plus petit.
18. Le gymnaste aux anneaux
a. Le poids du gymnaste est :
P = m . g = 78 × 10 = 780 N.
b. Tableau des caractéristiques des forces s’exerçant
sur le gymnaste :
3
P = 2 F1 3 = 2 × 4 000 x
= 4 618 N.
3
3
17. Tension des élingues
a. Tableau des caractéristiques des forces pour un
angle a = 60° :
Point
d’application
Droite
d’action
Sens
Valeur

P
G
Verticale
1 000 N

F1
De haut
en bas
E
580 N

F2
S
580 N
Dynamique des forces (échelle : 50%) :
Point
d’application
Droite
d’action
Sens
Valeur

P
G
Verticale
780 N

T1
De haut
en bas
A
400 N

T2
B
400 N
c. Dynamique des forces :
Graphiquement on trouve
T1 = T2 = 4 cm

Valeur de T1  :
4 × 100 = 400 N.

Valeur de T2  :
4 × 100 = 400 N.
T2
P
H
F1/60°
F1/45°
F1/30°
T1
75°
P
Horizontale
Graphiquement sur le dynamique des forces on
trouve
34 • CHAPITRE 8 - Comment soulever un objet ?
9 Comment soulever facilement
un objet ? HS1
Les activités
■■Activité 1 Quelle est l’influence du bras de levier d’une force ?
•Matériel
Un point de fixation
Un ressort
Un axe de rotation
Une barre à trous
Un repère
Un dynamomètre
•Réponses aux questions
1. Lorsque l’équilibre de la barre est réalisé, le produit F.d est constant. Son unité est le newton-mètre.
2. Lorsque le bras de levier augmente, la valeur de la force diminue.
3. En exerçant la même force, Julie
soulève le meuble dans le cas C, car cette situation présente le plus

grand bras de levier de la force F . Plus le bras de levier est grand, plus on peut soulever facilement un
meuble.
■■Activité 2 Comment calculer le moment d’un couple de forces ?
•Matériel pour un poste
Un point de fixation
Un ressort
Un axe de rotation
Une barre à trous
Un repère
Deux dynamomètres
•Réponses aux questions
1. Lorsque les couples de forces ont le même effet sur l’équilibre, leurs moments sont égaux.
2. La position de l’axe de rotation n’intervient pas dans le calcul du moment d’un couple de forces.
3. Lorsque les bras de levier d des couples de forces augmentent, les valeurs F des forces diminuent.
Pour des moments de couples égaux, les grandeurs F et d sont inversement proportionnelles.
CHAPITRE 9 - Comment soulever facilement un objet ? • 35
Les exercices
Tester ses connaissances
Q.C.M.
1 : A et C ; 2 : C ; 3 : A, B et C ; 4 : A ; 5 : A.
Situations
Tournevis
Tirebouchon
Cardan
Sens de rotation
ou
∙
⤺
∙
Moment du
couple ℳc
0,120 N.m
0,40 N.m
2,4 N.m
∙ ⤺
Tester ses capacités
1. À moment constant
a. Les forces ont le même effet sur la barre parce que
leurs moments sont égaux.
b. Lorsque le bras de levier diminue, la force augmente.
5. Arrache-clou
a.  F/O = F.OB = 15 × 0,80 = 12 N.m.
  = 0 N.m (la droite d’action de la force passe
R /O
par l’axe).
 F’/O = F ’.OA = 120 × 0,10 = 12 N.m.
2. Moment d’une force
 F/O = F.d,  F/O
a. Lorsqu’on utilise la relation

b. On a  F/O =   
(N.m) est le moment de F par rapport au point O,
F (N) représente la valeur de la force et d (m) est la
distance de la droite d’action de la force à l’axe (bras
de levier).
b. Le tableau suivant :
Situations
Barrière
de parking
Treuil
Cric
Valeur de la force
P = 500 N
P = 100 N
F = 20 N
Sens de rotation
ou
⤺
∙
∙
Mesure du bras de
levier
OG
= 0,30 m
OT
= 0,1 m
OA
= 35 cm
Moment de la force par
rapport à l’axe
150 N.m
10 N.m
7 N.m
∙ ⤺
. �����������������������������
Cette égalité traduit l’équi-
6. Mauvaise posture
Inventaire des moments des forces par rapport à O :
 P/O = P.OB = 300 × 0,6 = 180 N.m ; Sens ∙
 f/O = f1.OC = 250 × 0,15 = 37,5 N.m ; Sens ⤺
1
 R/O = 0 N.m
 F/O = F1.OA = 2 850 × 5 × 10-2 = 142,50 N.m ;
1
Sens ⤺
À l’équilibre, on a  P/O =   f/O +   F/O .
1
1
Appliquer le cours
7. Couple de serrage
3. Quel est le couple ?
a. On a un couple de forces dans le cas c.
b. Les autres schémas ne représentent pas un couple
de forces :
- en a, les forces n’ont pas la même valeur ;
- en b, les forces ont le même sens ;
- en d, les forces n’ont pas des droites d’action
parallèles.
4. Moment d’une couple de forces
a. Le couple de serrage correspond au moment
maximal de la force exercée par la main sur le manche de la clé dynamométrique.
b. Valeur de la force maximale :
 F/O
5,2
F = 
= 
= 28,9 N.
0,18
d
8. Système de 
freinage
a. Moment de F par rapport à O :
a. Lorsqu’on utilise la relation ℳC = F.d,
- ℳC (N.m) est le moment d’un couple de forces

 
( F1 ,F2 ),
- F (N) la valeur commune des deux forces,
- d (m) la distance séparant les droites d’action des
deux forces.
b. Le tableau suivant :
Situations
Tournevis
Tirebouchon
Distance AB
6 mm
8 cm
0,06 m ou
6 cm
Valeur de
 la
20 N
5N
40 N
force F
F ’/O
libre du pied de biche.
Cardan
 F/O = F.d = 40 × 0,27 = 10,8 N.m.


b. À l’équilibre, on a  F/O =   F''/
: F " est la force
O
exercée par le
liquide
de frein sur le piston.

La valeur de F '' est :
 F/O
10,8
F"=
= 
= 360 N.
0,03
d'
Par interaction, on déduit que la valeur de la force
exercée par le piston sur le liquide de frein est également de 360 N : F ’ = 360 N.

F ' 360
c. On a
=
= 9 donc la valeur de la force F a
F
40
d
été multipliée par 9 (cela correspond au rapport
d'
des bras de levier).
36 • CHAPITRE 9 - Comment soulever facilement un objet ?
9. Pinces de forge
a. L’inventaire des moments par rapport à l’axe sur
une demi-pince est :

 F/O = F1 × 0,60 = 30 N.m ;  F
= F ’2 × 0,05 ;
’2 /O
1
 R/O = 0.
30

On a  F/O =   F
soit F ’2 = 
= 600 N.
’2 /O
1
0,05
La valeur de la force exercée par une mâchoire est de
600 N.

 
b. ( F1 ,F2 ) ne constituent pas un couple de forces, car
ces deux forces n’ont pas des droites d’action distinctes.
13. Porte-personne

a.
 Distance à l’axe : pour P , on a OB = 1,40 m et pour
F on a OA = 0,40 m. 
b. Valeur de la force F :
 P/O =   F/O donne 600 × 1,40 = F × 0,40
soit F = 
Utiliser ses connaissances
840
= 2 100 N .
0,40
14. Conducteur d’engin de chantier (situation-
10. Centre de gravité


a. Moment de F1 par rapport à O :
problème)
 F/O = 320 × 1,80 = 576 N.m ; sens ⤺.
1
En appliquant la condition d’équilibre on a :

 F/O =  P/O . (Le moment de F2 par rapport à O

1
est nul, car la droite d’action de F2 passe par O).

Le moment de P par rapport à O est égal à : 576 N.m ;
Sens ∙.
b. Soit d la distance séparant le point O de la verticale
passant par G.

Le moment de P par rapport à O est :
 P/O = P.d = 800.d .
Plaçons le point B à l’intersection de la droite d’action


du poids P1 et du sol et calculons les moments des
forces exercées sur le tracteur par rapport à A.


- le moment de P1 est :  P/ A = m1.g.AB
1
= 2 300 × 10 × 1,6 = 36 800 N.m (sens ⤺) ;

- le moment de P2 est :  P/ A = m2.g.AC
2
= 2 500 × 10 × 1,5 = 37 500 N.m (sens ∙) ;
- en A la force de contact exercée par le sol sur la
roue a un moment nul.
 P/ A <   P/ A  : le chargeur de rochers bascule
1
567
On a 800.d = 576 soit d = 
= 0,71 m.
800
En position debout, la distance du sol au centre de
gravité du sportif est de : 1,80 – 0,71= 1,09 m.
11. Levage avec palan
a. Nombre de brins n = 2 ; P = 5 N ; F = 2,5 N.
On retient la relation F = 
Le rapport des deux valeurs des forces est :
2850
= 2,1.
1350
Dans cette étude, la force exercée sur le bas de
colonne est 2,1 fois plus faible lorsqu’on a les jambes
pliées que lorsqu’elles sont tendues.
P
5
, car 2,5 =  .
n
2
b. On a : n = 6 brins.
c. Poids de la charge : P = m.g = 60 × 10 = 600 N.

P
600
La valeur de la force F est : F = 
=
= 100 N.
6
6
2
autour du point A.
15. Dans la scierie (situation-problème)
Ce problème ouvert a de nombreuses solutions ; on
écarte celle avec la planche, car plus le bras de levier
est grand, plus on soulève facilement la scie (d’autres
arguments sur la flexibilité et la fragilité de la planche
peuvent être avancés).
a. Première solution : en position d’équilibre instable (les verticales passant par les centres de gravité de la scie et d’Arthur passent par les extrémités
du madrier). Il s’agit de déterminer la distance OB à
laquelle on place le point d’appui O. On pose OB = d.
R
12. Manutention sans risque
a. À l’équilibre, l’inventaire des moments des forces
G
par rapport à O donne :
A
 P/O = P.OB = 300 × 0,35 = 105 N.m (sens ∙).
PA
 f/O = f1.OC = 250 × 0,15 = 37,5 N.m (sens ⤺).
C
1
 F/O = F2.OA = 105 – 37,5 = 67,5 N.m (sens ⤺).
2
 R/O = 0.

b. La valeur de la force F2 exercée sur le bas de la
colonne est :
F2 = 
 F/O
2
0, 05
=
67,5
= 1 350 N.
0,05
Ps
O
M
B
Pm
3,15
d
6,30
CHAPITRE 9 - Comment soulever facilement un objet ? • 37



On note : Pm le poids du madrier ; Ps le poids de la

scie ; PA le poids d’Arthur (g = 10 N/kg)
R
On a :  P/O = PA.g.d = 300d (sens ∙)
A
 P/O = m.g.(d – 3,15) = 400(d – 3,15) (sens ∙)
m
 P/O = ms . g.(6,30 – d) = 2 300 (6,30 – d) (sens ⤺)
s
 R/O = 0.
À l’équilibre  P/O =   P/O +  P/O donne
s
m
A
2 300(6,30 – d) = 300d + 400(d – 3,15)
3 000 d = 15 750 soit d = 5,25 m.
OB = 5,25 m et OC = 1,05 m.
b. Autre proposition : on peut améliorer la stabilité
en avançant le madrier vers la gauche de 15 cm et en
déplaçant Arthur de 10 cm par exemple : les verticales, en G et A, ne passent plus par les extrémités du
madrier.
A
G
C
O’
PA
M
Ps
B
Pm
0,10
0,15
3,15
6,30
d’
La même démarche que précédemment permettrait
de calculer la position du point d’appui. Il va sans
dire que l’exercice est difficile ; nous recommandons
de le résoudre lors d’une activité dirigée avec l’aide
du professeur. Cela pourra donner l’occasion de faire
réviser la résolution des équations du premier degré
à une inconnue.
10 Quels sont les composants d’une eau
minérale ? HS2
Les activités
■■Activité 1Comment identifier quelques ions présents
dans une eau minérale ?
•Matériel
Quatre tubes à essai sur un porte-tubes
Une coupelle
Une pipette simple
Un bouchon avec un tube à dégagement adapté
•Produits
Une solution de nitrate d’argent
Une solution de chlorure de baryum
Une solution d’oxalate d’ammonium
Une eau minérale (eau Hépar par exemple)
Du sulfate de cuivre anhydre
Une bouteille d’eau minérale gazeuse (eau Perrier par exemple)
De l’eau de chaux
•Réponse aux questions
1. Les ions identifiés dans l’eau minérale sont :
- les ions chlorure de formule Cl– (précipité blanc avec le nitrate d’argent) ;
- les ions sulfate de formule SO42– (précipité blanc avec le chlorure de baryum) ;
- les ions calcium de formule Ca2+ (précipité blanc avec l’oxalate d’ammonium).
2. L’eau minérale contient de l’eau, car le sulfate de cuivre anhydre est devenu bleu.
38 • CHAPITRE 10 - Quels sont les composants d’une eau minérale ?
3. Le dégagement gazeux trouble l’eau de chaux. L’eau minérale gazeuse contient du dioxyde de
carbone dissous.
4. Une eau minérale contient :
- de l’eau mise en évidence avec le sulfate de cuivre anhydre ;
- des ions en solution caractérisés avec des réactifs ;
- du dioxyde de carbone lorsqu’elle est gazeuse, mis en évidence avec l’eau de chaux.
■■Activité 2 Comment modéliser des molécules ?
•Matériel
Une boîte de modèles moléculaires
•Réponse aux questions
1. Dans une molécule, les atomes sont matérialisés par des boules colorées : une boule blanche
représente un atome d’hydrogène ; une boule noire un atome de carbone ; une boule rouge un atome
d’oxygène ; une boule jaune un atome de soufre ; une boule verte un atome de chlore ; une boule
bleue un atome d’azote.
2. Les liaisons covalentes entre deux atomes sont matérialisées par des tiges, une tige par liaison.
3. La molécule de dihydrogène (a) est composée de 2 atomes d’hydrogène unis par une liaison
covalente. Sa formule brute est H2.
La molécule de chlorure d’hydrogène (b) est composée de 1 atome d’hydrogène et de 1 atome de
chlore, unis par une liaison de covalence. Sa formule brute est HCl.
La molécule de méthane (c) est composée de 1 atome de carbone et de 4 atomes d’hydrogène ;
chaque atome d’hydrogène est uni à l’atome de carbone par une liaison de covalence. Sa formule
brute est CH4.
La molécule de dioxygène (d) est composée de 2 atomes d’oxygène unis par deux liaisons de
covalence. Sa formule brute est O2.
4. Un atome d’hydrogène réalise une liaison de covalence.
Un atome de chlore réalise une liaison de covalence.
Un atome de carbone réalise quatre liaisons de covalence.
Un atome d’oxygène réalise deux liaisons de covalence.
Les documents
■■Comment établir le modèle de Lewis d’une molécule ?
•Réponse aux questions
1. Il manque un électron à un atome d’hydrogène pour obéir à la règle du duet.
Il manque deux électrons à un atome d’oxygène pour obéir à la règle de l’octet.
2. Dans le modèle de Lewis de la molécule d’eau :
- le tiret reliant l’atome d’oxygène à un atome d’hydrogène représente une liaison de covalence
simple ;
- chaque atome d’hydrogène porte un doublet d’électrons et l’atome d’oxygène porte quatre doublets
d’électrons.
3. Dans le modèle de Lewis de la molécule de dioxygène :
- les deux tirets reliant les deux atomes d’oxygène représentent une double liaison de covalence ;
- chaque atome d’oxygène porte quatre doublets d’électrons.
CHAPITRE 10 - Quels sont les composants d’une eau minérale ? • 39
Les exercices
Tester ses connaissances
Q.C.M.
1 : A et C ; 2 : A et C ;
6 : B ; 7 : B .
3:B;
4 . B ;
8. L’ion hydrogénocarbonate
a. L’ion hydrogénocarbonate est composé d’un
5:C;
Tester ses capacités
1. Test des ions chlorure
a. On utilise le nitrate d’argent pour identifier les
atome d’hydrogène, d’un atome de carbone et de
trois atomes d’oxygène .
b. Cet ion porte une charge négative .
9. Formule des ions
Ion sodium •
Ion potassium •
• Cl–
Ion cuivre •
• K+
ions chlorure dans une eau minérale .
b. On obtient le précipité blanc n°3 .
2. Tests des ions calcium et des ions sulfate
a. Pour identifier les ions calcium, on utilise l’oxalate
d’ammonium ; pour identifier les ions sulfate, on utilise le chlorure de baryum .
b. Lors de ces deux tests, on obtient un précipité
blanc .
3. Test de reconnaissance de l’eau
Pour mettre en évidence la présence d’eau dans une
boisson, on verse quelques gouttes de boisson sur du
sulfate de cuivre anhydre qui devient bleu en présence d’eau .
4. Test de reconnaissance du dioxyde de
carbone
Pour mettre en évidence le dioxyde de carbone dissous dans l’eau de Badoit :
- on place un bouchon muni d’un tube à dégagement sur le goulot d’une bouteille d’eau de Badoit ;
- on plonge le tube à dégagement dans de l’eau de
chaux contenue dans un tube à essai ou dans un verre
à pied ;
- on agite la bouteille afin que le dégagement gazeux
soit plus important ;
- on observe le trouble de l’eau de chaux .
5. L’atome d’oxygène
—
Le modèle de Lewis de l’atome d’oxygène est | O •
•
6. La molécule d’eau
Le modèle de Lewis de la molécule d’eau est :
O
• Cu2+
Ion sulfate •
• Mg2+
Ion chlorure •
• SO2−
4
Ion magnésium •
• Na+
10. Ion calcium
Un atome de calcium qui a perdu deux électrons
devient un ion calcium de formule Ca2+ .
Appliquer le cours
11. Symbole des atomes
Les symboles incorrects sont : magnésium : Mg ;
sodium : Na ; potassium : K ; azote : N .
12. Structure de l’atome
a. Le symbole chimique du potassium est K, celui du
sodium est Na .
b. Le noyau d’un atome de potassium comporte 19
protons et 39 – 19 = 20 neutrons ; son nuage électronique contient 19 électrons .
Le noyau d’un atome de sodium comporte 11 protons et 23 – 11 = 12 neutrons ; son nuage électronique
contient 11 électrons .
13. Atome et ion magnésium
a. L’atome de magnésium possède 2 + 8 + 2 = 12 électrons au total .
b. Sa couche externe contient 2 électrons .
c. L’ion magnésium a pour formule Mg2+, car il provient d’un atome de magnésium ayant perdu les deux
électrons de sa couche externe M pour obéir à la règle
de l’octet .
14. Ion oxyde
a. La structure électronique de l’atome d’oxygène est
K2L6 .
H
H .
b. La couche externe L de l’atome d’oxygène contient
6 électrons .
7. Le saccharose
c. L’ion oxyde a pour formule O2–, car il provient d’un
Le saccharose est composé de 12 atomes de carbone,
de 22 atomes d’hydrogène et de 11 atomes d’oxygène .
atome d’oxygène ayant gagné deux électrons afin de
compléter sa couche externe L à huit électrons pour
satisfaire la règle de l’octet .
40 • CHAPITRE 10 - Quels sont les composants d’une eau minérale ?
15. L’ion carbonate
20. Modèles moléculaires compacts
La formule de l’ion carbonate est CO32– .
Eau : 5 ; dioxygène : 3 ; dioxyde de carbone : 2 ; dihydrogène : 4 ; chlorure d’hydrogène : 1 .
16. Structure électronique
Atome de sodium 23
Na •
11
• K2L8
Ion sodium Na+ •
• K2L1
Atome de chlore
35
17
Ion chlorure
21. Modèles de Lewis de quelques molécules
Molécules
Cl •
• K2L8M8
Cl–
• K2
•
Atome de lithium Li •
• K2L8M7
Ion lithium Li+ •
• K2L8M1
7
3
Modèles
de Lewis
17. Rechercher la formule d’un ion
monoatomique
Atome
Oxygène
16
8
O
Magnésium
24
12
Mg
Fluor
19
9
27
13
8
12
9
13
Structure
électronique
K2L6
K2L8M2
K2L7
K2L8M3
Nombre
d’électrons
sur la couche
externe
6
2
7
3
Formule
de l’ion
correspondant
O2–
Mg2+
F–
Al3+
CI
Dioxygène
O
Molécules
H
CI
Dioxyde de carbone
O
O
H
C
C
O
Ammoniac
Méthane
H
Modèles de
Lewis
CI
H
Molécules
Al
Nombre
d’électrons
O
H
Chlorure
d’hydrogène
Dichlore
Modèles de
Lewis
Aluminium
F
Eau
H
H
N
H
H
H
Utiliser ses connaissances
22. En plomberie
a.
18. Caractérisation des ions chlorure
Nom de l’élément
Cuivre
Étain
Symbole chimique
Cu
Sn
Nombre de protons
29
50
b. Il observe un précipité blanc dans les tubes à essai
Nombre d’électrons
29
50
contenant de l’eau de Vittel, de l’eau Volvic et de l’eau
du robinet .
Nombre de neutrons
34
70
a. Pour caractériser les ions chlorure, Cyril a utilisé
une solution de nitrate d’argent .
c. On trouve donc des ions chlorure dans les eaux
minérales de Vittel et Volvic ainsi que dans l’eau du
robinet .
d. On n’obtient pas de précipité avec l’eau déminéralisée, car elle ne contient pas d’ions en solution .
19. Composition d’une eau minérale
a. Formules des ions composant l’eau minérale :
sodium : Na+ ; potassium : K+ ; calcium : Ca2+ ; magnésium : Mg2+ ; bicarbonate : HCO3– ; chlorure : Cl– ; sulfate : SO42–; fluorure : F– .
b. On identifie les ions calcium avec l’oxalate d’ammonium, les ions chlorure avec le nitrate d’argent et
les ions sulfate avec le chlorure de baryum . Dans les
trois tests, on observe un précipité blanc .
c. Sur l’étiquette est mentionné que l’eau minérale
Saint-Yorre est une eau minérale naturelle gazeuse .
Le gaz dissous est du dioxyde de carbone pouvant
être identifié avec de l’eau de chaux qui se trouble en
sa présence .
b. Le pictogramme signifie que le décapant utilisé
par les plombiers est un produit corrosif .
23. Chromage
a. L’atome de chrome comporte autant d’électrons
que de protons, donc 24 électrons dans son nuage
électronique .
b. L’ion chrome, de formule Cr3+, provient d’un
atome de chrome qui a perdu 3 électrons ; l’ion
chrome comporte donc 24 protons dans son noyau et
21 électrons dans son nuage électronique .
24. Cellules photovoltaïques
a. La structure électronique du silicium est : K2L8M4 .
b. La couche externe M du silicium comporte 4 électrons .
c. Le modèle de Lewis de l’atome de silicium est
•
• Si • .
•
d. L’atome de silicium est tétravalent, car il possède
quatre électrons célibataires sur sa couche externe, il
est susceptible de créer quatre liaisons de covalence .
CHAPITRE 10 - Quels sont les composants d’une eau minérale ? • 41
25. Paludier
c. Les liquides contenant de l’eau sont : la limonade,
a. Structure électronique de l’atome de sodium :
le lait, le vinaigre blanc.
Le white spirit ne contient pas d’eau.
K2L8M1 ; celle de l’atome de chlore : K2L8M7.
b. L’atome de sodium perd un électron (1 électron
célibataire sur la couche M) pour saturer sa couche
externe à huit électrons (règle de l’octet) et donne
l’ion sodium de formule Na+.
c. L’atome de chlore gagne un électron (7 électrons
sur la couche M) pour saturer sa couche externe à
huit électrons (règle de l’octet) et donne l’ion chlorure de formule Cl–.
26. Jardinier
Les ions mis en évidence dans la bouillie bordelaise
sont :
- les ions cuivre II de formule Cu2+, car on a obtenu un
précipité bleu avec la soude ;
- les ions sulfate de formule SO42–, car lors du test avec
la solution de chlorure de baryum, on a obtenu un
précipité blanc.
28. Pneus gonflés au diazote
a. La représentation respectant la règle de l’octet est
N
N , car chaque atome d’azote comporte
quatre doublets d’électrons.
b. Le modèle de Lewis du diazote est donc
N
N .
c. Les deux atomes d’azote sont unis par une triple
liaison de covalence matérialisée par la mise en commun de trois doublets d’électrons.
29. Matières plastiques
a. La molécule d’éthylène comporte 2 atomes de carbone et 4 atomes d’hydrogène.
b. La formule brute de l’éthylène est C2H4.
c. Le modèle de Lewis de l’éthylène est
H
H
C
H
27. Liquide d’usage courant
C
.
H
a. On effectue le test de reconnaissance de l’eau avec
d. Chaque atome d’hydrogène est uni à un atome de
le sulfate de cuivre anhydre.
b. En contact avec de l’eau, le sulfate de cuivre anhydre (blanc) devient bleu.
carbone par une liaison de covalence simple.
Les atomes de carbone sont unis par une double
liaison de covalence.
11 Comment déterminer la concentration
d’une substance dans une solution ? HS2
Les activités
■■Activité 1 Comment préparer une boisson sucrée par dissolution ?
•Matériel
Une balance électronique à 0,1 g
Une capsule
Une spatule
Un entonnoir
Une fiole jaugée de 100 mL
Une pipette simple
•Produits
Du saccharose en poudre - Une pissette d’eau distillée
•Réponse aux questions
1. On a dissous 1,7 g de saccharose dans 100 mL d’eau distillée.
2. La masse de saccharose que contiendrait un litre de solution est de 17 g.
3. La concentration est la masse d’une substance dissoute par litre de solution. Dans notre exemple,
on dit que la concentration massique de la solution est de 17 g/L.
42 • CHAPITRE 11 - Comment déterminer la concentration d’une substance dans une solution ?
■■Activité 2Comment déterminer le caractère acide, basique
ou neutre d’une solution ?
•Matériel
Trois capsules
Trois petits béchers
Trois tubes à essai
Un agitateur de verre
Du papier pH
Un pH-mètre, une pissette d’eau, du papier filtre fin (papier Joseph)
•Produits
Eau minérale Volvic
Vinaigre blanc
Liquide pour lave-vaisselle
Solution de bleu de bromothymol
•Réponse aux questions
1. 2. Classement des solutions par ordre de pH croissant : vinaigre blanc (pH = 2,1) ; eau de
Volvic (pH = 7) ; liquide pour lave-vaisselle (pH = 13).
3. Avec le bleu de bromothymol :
- une solution acide devient jaune ;
- une solution basique devient bleue ;
- une solution neutre devient verte.
Les documents
■■Les dangers des liquides d’usage courant
•Réponse aux questions
1. Avant d’utiliser le Destop, il faut lire attentivement l’étiquette portée sur le flacon afin de s’informer
sur les dangers et les précautions à prendre lors de son utilisation.
2. a. Le pictogramme indique que le Destop est un produit corrosif.
b. L’étiquette du produit Destop mentionne que le produit est corrosif, elle informe l’utilisateur
qu’il faut éviter le contact du produit avec la peau et les yeux, car il provoque de graves brûlures. Les
dangers indiqués sur l’étiquette sont donc en accord avec ceux indiqués par le pictogramme.
3. Le Destop contient de l’hydroxyde de sodium, produit très corrosif et dangereux.
4. Pour utiliser ce produit, il faut porter un vêtement de protection approprié, des gants et un appareil
de protection des yeux et du visage.
5. Le Destop dissout intégralement les matières organiques, les cheveux, les graisses.
Les exercices
Tester ses connaissances
Tester ses capacités
Q.C.M.
1. B et C ;
5. C ; 6. C ;
1. Saccharose
2. C ;
7. A et C ;
3. B ;
8. B ;
4. B ;
9. B.
M(C12H22O11) = 12 × M(C) + 22 × M(H) + 11 × M(O)
= 12 × 12 + 22 × 1 + 11 × 16 = 342 g/mol.
La masse molaire moléculaire du saccharose est de
342 g/mol.
CHAPITRE 11 - Comment déterminer la concentration d’une substance dans une solution ? • 43
2. Café sucré
10. Concentration molaire
La concentration massique de la solution de café est
égale à 50 g/L.
a. La formule permettant de calculer la concentra-
3. Boisson sucrée
b. C : en mol/L ; n : en mol ; V en L.
La concentration molaire de la boisson sucrée est de
0,1 mol/L.
11. Quantité de matière
4. Composition d’une eau minérale
L’étiquette indique qu’un litre d’eau d’Evian contient
80 mg d’ions calcium.
Quantité de matière d’ions calcium contenus dans
m 0,080
80 mg : n =
=
≈ 0,002 mol.
M
40,1
La concentration molaire des ions calcium de l’eau
d’Evian est donc égale à 0,002 mol/L.
5. Sirop de menthe
a. Cette opération est une dilution.
b. Un litre de sirop de menthe contient 3 moles de
saccharose dissous, donc 0,01 L de sirop contient :
0,03 mole de saccharose.
La concentration molaire de la boisson préparée est
n 0,03
=
= 0,2 mol/L.
C=
V 0,15
6. Dilution de l’eau de javel
a. On doit prélever 10 mL d’eau de javel pour réaliser
100 mL d’une solution diluée 10 fois.
b. Pour effectuer cette dilution, il faut utiliser une
pipette jaugée de 10 mL.
tion molaire C d’une solution est : C =
n
.
V
n = C . V = 0,5 × 0,1 = 0,05 mol.
La quantité de matière de l’espèce chimique dissoute
est de 0,05 mol.
12. Dissolution
m = Cm . V = 400 × 0,5 = 200 g.
Il faut 200 g de sucre pour préparer le bocal de fruits
au sirop.
13. Jus d’orange
Quantité de matière de sucre dans 18 g de sucre :
m
18
n 0,1
n=
=
= 0,1 mol. C =
=
= 1 mol/L.
M sucre
180
V 0,1
La concentration molaire du sucre dans le jus de
fruits est de 1 mol/L.
14. Solution de saccharose
n = C . V = 0,25 × 0,5 = 0,125 mol.
D’où une masse m = n.M = 0,125 × 342 = 42,7 g.
Il faut peser 42,7 g de saccharose pour préparer 500 ml
d’une solution de saccharose de concentration :
C = 0,25 mol/L.
15. Lire l’étiquette d’une eau minérale
a. Les valeurs chiffrées sur l’étiquette indiquent les
7. Acide ou basique ?
La comparaison de la couleur (rouge orangée) prise
par le papier pH au nuancier de la boîte, indique que
le pH de la solution est de 2.
8. Lire l’étiquette d’un produit ménager
a. Le pictogramme indique que le produit ménager
est nocif.
b. L’étiquette mentionne que le CANARD WC ANTITARTRE contient de l’acide chlorhydrique.
c. Il ne faut pas mélanger le produit avec de l’eau de
javel, car du dichlore, un gaz dangereux, peut se libérer.
d. Il faut éviter le contact avec la peau et les yeux, car le
produit est irritant et provoque des lésions oculaires.
Appliquer le cours
9. Grandeurs et unités
concentrations massiques des ions en solution dans
l’eau minérale St-Yorre.
b. Sur l’étiquette, on lit : Sodium 1 708 mg/L.
Dans 15 cL d’eau minérale, la masse de sodium est :
0,15 × 1 708 = 256,2 mg.
On absorbe donc 256,2 mg de sodium lorsqu’on boit
un verre de 15 cL d’eau minérale St-Yorre.
m
1,708
c. n(Na+) =
=
= 0,074 mol ;
M(Na)
23
m
0,322
n(Cl–) =
=
= 9 ×10–3 mol.
M(Cl) 35,5
Un litre d’eau minérale St-Yorre contient 0,074 mol
d’ions sodium et 9×10–3 mol d’ions chlorure.
d. Le sel est un composé ionique de chlorure de
sodium. Cette eau riche en ions sodium et en ions
chlorure a donc un goût salé.
Unités
16. Dureté d’une eau minérale
Masse •
• mol/L
a. L’étiquette indique les concentrations massiques
Volume •
• g/mol
en ions Ca2+ et en ions Mg2+ :
C(Ca2+) = 203,8 mg/L et C(Mg2+) = 43,1 mg/L.
Grandeurs
Quantité de matière •
Masse molaire •
Concentration massique •
Concentration molaire •
•g
• mol
•L
• g/L
C(Ca 2+ ) C(Mg 2+ )
203,8 43,1
+
=
+
4
2,4
4
2,4
≈ 68,9.
La dureté de l’eau de Vittel est donc de 68,9 °HT.
Dureté : D =
44 • CHAPITRE 11 - Comment déterminer la concentration d’une substance dans une solution ?
b. L’eau de Vittel est une eau dure, car sa dureté est
c. - L’eau de Badoit est la plus minéralisée, car son
supérieure à 15 °HT.
c. L’utilisation abusive de l’eau de Vittel dans une
cafetière électrique entraîne un entartrage (dépôt de
calcaire) du circuit d’eau chaude.
résidu à sec est le plus grand (1 200g/L).
- L’eau de Badoit est plus dure que l’eau de Volvic,
car les concentrations en ions calcium et magnésium
dans l’eau de Badoit sont plus élevées.
- L’eau de Badoit (pH = 6) est plus acide que l’eau de
Volvic (pH = 7), car son pH est plus faible.
17. Dilution d’un sirop de fraise
a. Le volume de liquide dans le verre est
V = 180 + 20 = 200 mL.
23. Réaliser une dissolution au laboratoire
b. Le sirop a été dilué 10 fois.
c. 700 × 0,02 = 14 g.
a. Masse de pastilles d’hydroxyde de sodium à peser :
20 mL de sirop contiennent donc 14 g de sucre ; le
verre contient 14 g de sucre.
18. pH de quelques solutions
a. Coca-Cola (pH = 2,5) : acide ; Jus de citron (pH =
3,1) : acide ; eau de Vichy (pH = 8,2) : basique ; eau de
Perrier (pH = 5,0) : acide ; lait (pH = 7,0) : neutre.
b. La solution la plus acide est le Coca-Cola ; la plus
basique est l’eau de Vichy.
19. Acidité des boissons
a. L’acidité de la boisson au Coca est due aux acidifiants : acide orthophosphorique et acide citrique.
b. L’acidité de cette boisson peut être vérifiée en
mesurant son pH avec un pH-mètre.
20. Ions et pH
a. La solution 1 est la plus acide : elle contient donc
davantage d’ions hydrogène H+ que d’ions hydroxyde
HO–.
b. La solution 3 est basique : elle contient davantage
d’ions hydroxyde HO– que d’ions hydrogène H+.
c. La solution 2 est neutre : elle contient autant d’ions
hydrogène H+ que d’ions hydroxyde HO–.
Utiliser ses connaissances
21. Sportif
m 20,1
=
= 67 g/L.
V
0,3
La concentration massique des glucides dans la boisson énergétique est donc de 67 g/L.
a. Cm =
m 0,21
=
= 0,7 g/L.
V
0,3
La concentration massique du sodium dans la boisson énergétique est donc de 0,7 g/L.
b. Cm =
22. Comparer des eaux de boisson
a. L’eau de Badoit et l’eau de Volvic n’ont pas les
mêmes propriétés, car leurs teneurs en sels minéraux
sont différentes ainsi que leurs pH.
b. Le résidu à sec à 180 °C indique la teneur globale
(ou minéralisation totale) en sels minéraux en mg/L
après chauffage à 180°C.
m = C.M.V = 0,5 × 40 × 0,1 = 2 g.
Pour préparer la solution, il faut dissoudre 2 g de pastilles d’hydroxyde de sodium.
b. Matériel de chimie nécessaire : une balance électronique, une capsule, une spatule, un entonnoir, une
fiole jaugée de 100 mL, une pissette d’eau distillée.
24. Hygiène
a. Masse molaire moléculaire de l’hypochlorite de
sodium :
M(NaClO) = 23 + 35,5 + 16 = 74,5 g/mol.
b. Quantité de matière dans 4,25 g d’hypochlorite de
sodium :
m
4,25
=
≈ 0,06 mol.
M
74,5
n 0,06
Volume d’eau nécessaire : V =
=
= 6 L.
C 0,01
Il faut 6 litres d’eau pour dissoudre une pastille de
masse m égale à 4,25 g.
n=
25. Lessive (développement durable)
a. Les doses de lessive à choisir dépendent à la fois
de la dureté de l’eau et de la saleté du linge à laver.
Avec une eau dure, il faut davantage de lessive pour
nettoyer le linge qu’avec une eau douce.
b. Une eau dure contient beaucoup d’ions calcium
et d’ions magnésium. Sa dureté est supérieure à
15 °HT.
c. La dureté d’une eau dépend de la nature géologique des terrains qu’elle a traversés. Ainsi, un sol
crayeux ou calcaire donnera une eau dure, riche en
ions calcium, alors qu’un sol granitique ou sablonneux donnera plutôt une eau douce.
d. La dureté de l’eau se constate principalement :
- par la formation de dépôts insolubles avec les savons
(« l’eau ne mousse pas ») dans les canalisations ;
- par des taches grisâtres sur le linge ;
- par l’entartrage (couche de calcaire) des tuyaux
d’eau chaude.
e. Les lessives contiennent des substances toxiques
pour l’environnement. Les eaux de lessive sont difficiles à dégrader dans les stations d’épuration. Ces
stations n’existent pas partout, et en cas de rejet dans
le sol, elles peuvent polluer la nappe phréatique.
CHAPITRE 11 - Comment déterminer la concentration d’une substance dans une solution ? • 45
12 Que contient un soda ? HS2
Les activités
■■Activité 1Comment identifier le glucose et les colorants contenus dans
une boisson ?
Expérience 1
•Matériel
Un bécher
Deux tubes à essai
Une plaque chauffante
•Produits
Une solution de glucose
Une boisson au Coca-Cola
Une solution de liqueur de Fehling
Expérience 2
•Matériel
Un bécher recouvert par une capsule ou une boîte de Pétri
Une feuille de papier filtre ou de papier à dessin (5 cm × 3 cm)
Une règle, un crayon à papier
Tubes effilés ou piques en bois
•Produits
Une solution d’eau salée
Du sirop de menthe
Une solution aqueuse de tartrazine (E 102)
Une solution aqueuse de bleu patenté (E 131)
•Réponse aux questions
1. Expérience 1 - Dans les deux tubes, la liqueur de Fehling a viré au rouge brique. Le Coca Cola
contient donc du glucose.
2. Expérience 2
a. On observe une migration de taches vers le haut de la feuille. Les taches jaunes migrent plus
facilement et atteignent la même hauteur, les taches bleues migrent moins.
b. Le sirop de menthe donne deux taches : une bleue et une jaune.
c. On compare les positions des taches obtenues en mesurant les distances parcourues par chacune
d’elles.
3. On identifie le glucose contenu dans un soda ou un sirop avec le test de la liqueur de Fehling : la
liqueur de Fehling vire au rouge brique en présence de glucose.
La chromatographie permet d’identifier les colorants.
46 • CHAPITRE 12 - Que contient un soda ?
■■Activité 2 Comment extraire un arôme ?
Expérience 1
•Matériel
Un couteau à bout rond
Un mixeur
Un erlenmeyer avec un bouchon
Une éprouvette graduée de 25 mL
Un dispositif de filtration sous pression réduite (fiole à vide, trompe à eau, support, filtre Büchner,
joint conique, papier filtre…)
Une ampoule à décanter
Deux béchers
•Produits
Une orange
Du cyclohexane
De l’eau distillée
Expérience 2
•Matériel
Un couteau à bout rond
Deux supports avec noix de serrage, pinces
Un élévateur à croisillons
Un chauffe-ballon thermostaté
Un ballon de 250 mL
Une tête de colonne, un condenseur à eau, une allonge coudée
Une éprouvette graduée de 50 mL
Papier filtre
•Produits
Eau distillée
Une orange
•Réponse aux questions
1. Expérience 1
a. Pour extraire les arômes des zestes d’orange, on broie et on mélange les zestes dans un solvant, ici
le cyclohexane dans lequel les arômes à extraire sont très solubles.
b. Le cyclohexane est un liquide inflammable ; il doit être manipulé à l’écart de toute flamme ou de
toute source de chaleur.
c. d(cyclohexane) < d(eau), la phase organique contenant le cyclohexane se trouve au-dessus de la
phase aqueuse.
d. Pour récupérer l’essence de limonène contenue dans le cyclohexane, il suffit de placer le bécher
contenant la phase organique sous une hotte aspirante afin que le cyclohexane s’évapore, car il est
très volatil.
2. Expérience 2
a. Les vapeurs d’eau chargées des arômes de zestes d’orange se condensent dans le tube central du
réfrigérant, refroidi par un courant d’eau froide. Le distillat est recueilli à la sortie du réfrigérant.
b. Le distillat présente deux phases : de l’eau et le limonène.
c. Le limonène se trouve dans la phase supérieure. On utilise une ampoule à décanter pour séparer les
deux phases et récupérer le limonène.
CHAPITRE 12 - Que contient un soda ? • 47
Les exercices
Tester ses connaissances
Q.C.M.
1 : B et C ;
5 : A et C ;
2 : A et B ;
6. : B et C ;
3 : A et C ;
7 : B et C.
h
18
=
≈ 0,38.
H
48
h
21
b. Rapport frontal du colorant : Rf =
=
≈ 0,38.
H
55
On trouve le rapport frontal du colorant E 131.
Le sirop contient du bleu patenté. Sa couleur est
bleue patenté.
colorant E 131 : Rf =
4 : C ;
Tester ses capacités
1. Connaître le matériel de chromatographie
a : cuve à chromatographie ; b : front de l’éluant ; c :
phase fixe ; d : ligne de dépôt ; e : éluant ; f : couvercle.
7. Ampoule à décanter
a. Une ampoule à décanter est utilisée pour extraire
2. Analyser un chromatogramme
une substance par solubilisation dans un solvant. Elle
permet de recueillir la phase organique contenant le
solvant et l’espèce à extraire.
b. 1er cas : le cyclohexane, moins dense, surnage.
Le chromatogramme obtenu est le c, car le colorant
bleu, plus soluble que le colorant rouge, migre plus
haut. Les taches bleues migrent à la même hauteur,
ainsi que les taches rouges.
cyclohexane
3. Rapport frontal
eau
a. h1 = 8 mm ; h2 = 18 mm ; h3 = 26 mm ; H = 31 mm.
b. Les rapports frontaux sont :
h1
h
8
18
=
≈ 0,26 ; R(C2) = 2 =
≈ 0,58 ;
31
31
H
H
h
26
R(C3) = 3 =
≈ 0,84.
31
H
R(C1) =
2e cas : le dichlorométhane, plus dense, est situé dans
la partie inférieure.
Appliquer le cours
4. Test du glucose
a. On utilise la liqueur de Fehling pour mettre en évidence la présence de glucose dans une solution.
b. Dans le tube 1, on a versé une solution sucrée
contenant du glucose, car la liqueur de Fehling a viré
au rouge brique.
5. Réaliser une chromatographie
eau
dichlorométhane
a. La phase fixe est la feuille de papier à chromato-
8. Choix d’un solvant et sécurité
graphie.
b. L’éluant est le mélange de cyclohexane et d’éther.
Lors de sa migration sur le support fixe, l’éluant
entraîne les constituants de l’huile essentielle de
peaux d’oranges.
c. Le trait (a) matérialise le front de l’éluant, le trait
(b) la ligne de dépôt.
d. L’huile essentielle de peaux d’oranges contient du
limonène et du citral, car les deux taches correspondant au dépôt n° 4 migrent à la même hauteur que les
taches correspondant aux dépôts n° 1 de limonène et
n° 3 de citral.
a. Le composé C, non miscible à l’eau, ne peut pas se
6. Rapport frontal
a. Rapports frontaux :
colorant E 102 : Rf =
h
35
= ≈ 0,73 ;
H
48
48 • CHAPITRE 12 - Que contient un soda ?
mélanger avec l’eau, il forme un mélange hétérogène
avec l’eau.
b. Le solvant ne doit pas être miscible avec l’eau : on
écarte l’éthanol, on a le choix entre le cyclohexane et
le dichlorométhane.
Le composé C doit être très soluble dans le solvant : le
solvant ne peut donc pas être le cyclohexane. Le solvant choisi est donc le dichlorométhane.
c. Les pictogrammes apposés sur le flacon de dichlorométhane signifient que le solvant est irritant et
inflammable.
Pour éviter une réaction au contact avec la peau, les
yeux et les voies respiratoires, il faut porter des gants
de protection, des lunettes et manipuler le dichlorométhane sous une hotte aspirante.
Comme ce solvant est inflammable, il faut le conserver et l’utiliser à l’écart de toute flamme ou de source
de chaleur.
d. Le dichlorométhane, plus dense que l’eau, se situe
sous la phase aqueuse.
eau
dichlorométhane
Le Coca Cola classique contient de l’eau, du glucose
et du dioxyde de carbone.
c. Le Coca Cola est qualifié de Light lorsqu’il ne
contient pas de glucose.
d. Boire régulièrement des boissons trop sucrées
peut être néfaste pour la santé en raison du risque de
diabète et d’obésité que ces boissons peuvent entraîner.
11. Chromatogramme de boissons
a. Les rapports frontaux sont :
38
≈ 0,88 ;
43
22
- acide citrique : Rf =
≈ 0,51 ;
43
24
- acide malique : Rf =
≈ 0,56 ;
43
15
- acide tartrique : Rf =
≈ 0,35.
43
b. Les acides citrique et malique ont des rapports
frontaux voisins et sont difficilement différentiables.
c. Le jus est le n° 3 : il contient de l’acide tartrique et
de l’acide malique mais pas d’acide lactique.
d. Les vins contiennent tous de l’acide lactique. Le
vin n° 2 est celui dont la fermentation est la plus avancée, car tout l’acide malique a été transformé en acide
lactique.
- acide lactique : Rf =
9. Principe de l’hydrodistillation
a. L’hydrodistillation est un procédé d’extraction et
de séparation de certaines substances organiques par
entraînement à la vapeur d’eau.
b. 1 : Mélange de zestes d’orange et d’eau ; 2 : chauffeballon ; 3 : ballon ; 4 : thermomètre ; 5 : eau ; 6 : réfrigérant ; 7 : éprouvette ; 8 : distillat ; 9 : eau ; 10 : huile
essentielle.
c. La vapeur d’eau produite au cours de l’ébullition
entraîne les substances à extraire non solubles dans
le liquide vaporisé (ici, l’eau).
d. Le réfrigérant condense les vapeurs et fournit un
distillat, mélange d’eau de condensation et d’huile
essentielle contenant le limonène à extraire.
e. L’huile essentielle contenant le limonène est
recueillie dans l’éprouvette à la sortie du réfrigérant.
Utiliser ses connaissances
10. Au distributeur de boissons
a. Test de présence de l’eau : on verse quelques gouttes de chaque boisson sur du sulfate de cuivre anhydre contenu dans une capsule ou une boîte de Pétri.
Si le sulfate de cuivre anhydre devient bleu, la boisson
contient de l’eau.
Test du glucose : dans un tube à essai, on chauffe
au bain-marie quelques millilitres de chaque boisson avec 1 à 2 millilitres de liqueur de Fehling. Si le
mélange prend une couleur rouge brique, la boisson
contient du glucose.
Test du dioxyde de carbone : faire barboter dans de
l’eau de chaux le gaz dissous dans chaque boisson. Si
l’eau de chaux devient trouble, la boisson contient du
dioxyde de carbone.
b. Le Coca Cola Light contient de l’eau et du dioxyde
de carbone.
12. Chromatographie des glucides
a. La phase fixe est la plaque chromatographique,
elle est composée d’une couche de silice hydratée sur
une feuille d’aluminium.
b. La phase mobile est l’éluant, mélange composé
d’éthanol, d’acide éthanoïque et de butanone.
c. On a révélé la plaque chromatographique afin de
colorer les taches de sucre initialement invisibles.
d. Les rapports frontaux sont :
20
- glucose G : Rf =
≈ 0,37 ;
54
6
- saccharose S: Rf =
≈ 0,11 ;
54
16
- fructose F : Rf =
≈ 0,30.
54
e. Le jus d’orange est composé de glucose et de saccharose.
f. L’étiquette indique que le jus d’orange contient
du saccharose et du sirop de glucose, ce qui est en
accord avec les résultats obtenus avec la chromatographie du jus d’orange.
CHAPITRE 12 - Que contient un soda ? • 49
13 Que se passe-t-il lorsque des produits
d’usage courant réagissent entre eux ? HS2
Les activités
■■Activité 1Quelle réaction se produit-il entre l’acide chlorhydrique
et le calcaire ?
•Matériel
Deux tubes à essai
Un tube à dégagement
Un verre à pied
Un rouleau de papier indicateur de pH
Une coupelle
•Produits
Acide chlorhydrique dilué
Une solution de nitrate d’argent
Des morceaux de calcaire
Eau de chaux
Expérience 1
Elle permet d’identifier les ions contenus dans l’acide chlorhydrique afin de pouvoir écrire sa formule
ionique. L’identification de l’ion chlorure Cl– a déjà été réalisée au chapitre 10, l’étude de l’acidité a
été revue au chapitre 11.
Expérience 2
Elle permet d’identifier le gaz qui se dégage lors de la réaction entre l’acide chlorhydrique et le
calcaire. Cette réaction a été déjà vue au collège en cours de SVT pour reconnaître les roches calcaires.
•Réponses aux questions
Expérience 1
1. Le pH de l’acide chlorhydrique est voisin de 2. L’ion responsable de l’acidité est l’ion H+.
2. L’ion chlorure est identifié par le test au nitrate d’argent.
3. La formule ionique de l’acide chlorhydrique est : (H+ + Cl–).
Expérience 2
4. Il se produit entre l’acide chlorhydrique et le tartre la même réaction qu’entre l’acide chlorhydrique
et le calcaire. L’acide chlorhydrique attaque le tartre avec dégagement de dioxyde de carbone qui
trouble l’eau de chaux.
■■Activité 2 Quelles sont les caractéristiques d’une réaction chimique ?
•Matériel
Une bouteille d’eau minérale de un litre avec son bouchon
Une balance électronique
•Produits
Acide chlorhydrique dilué
Des morceaux de calcaire
Expérience 1
Elle permet de vérifier que les atomes se conservent au cours d’une réaction chimique.
50 • CHAPITRE 13 - Que se passe-t-il lorsque des produits d’usage courant réagissent entre eux ?
Expérience 2
La bouteille avec son bouchon est pesée avec l’acide chlorhydrique et le calcaire. Cette réaction
permet de vérifier la conservation de la masse.
•Répondre aux questions
Expérience 1
1. Les atomes contenus dans les réactifs sont les atomes d’hydrogène, de chlore, de calcium, de
carbone et d’oxygène.
2. Les atomes contenus dans les produits sont les mêmes mais ils sont disposés différemment.
Les atomes se conservent au cours de la réaction chimique.
Expérience 2
3. La bouteille gonfle, car il se dégage du dioxyde de carbone qui ne peut pas s’échapper.
4. La masse des réactifs et des produits se conserve au cours de la réaction chimique. La masse des
réactifs diminue mais la masse des produits augmente si bien que la masse totale ne change pas.
Documentation
■■Risques et sécurité
1. La concentration de l’acide étant inférieure à 25 %, les risques encourus par sa manipulation sont :
- R36 irritant pour les yeux.
- R37 irritant pour les voies respiratoires.
- R 38 irritant pour la peau.
2. Les principaux moyens de sécurité à mettre en œuvre sont :
- S24 Éviter le contact avec la peau ;
- S25 Éviter le contact avec les yeux ;
- S26 En cas de contact avec les yeux, laver immédiatement et abondamment avec de l’eau et
consulter un spécialiste ;
- S28 Après contact avec la peau, se laver immédiatement et abondamment avec de l’eau ;
- S36/37/39 Porter un vêtement de protection approprié, des gants et un appareil de protection des
yeux et du visage.
Les exercices
Tester ses connaissances
Q.C.M.
1. : B ;
6. : B.
2. : A ;
3. : C ;
4. : C ;
5. : A ; Tester ses capacités
1. Dilution de l’acide chlorhydrique
a. La concentration de l’acide dilué étant inférieure à
25 %, les risques encourus par sa manipulation sont :
- R36 irritant pour les yeux ;
- R37 irritant pour les voies respiratoires ;
- R 38 irritant pour la peau.
b. Les principaux moyens de sécurité à mettre en
œuvre sont :
- S24 éviter le contact avec la peau ;
- S25 éviter le contact avec les yeux. ;
- S26 en cas de contact avec les yeux, laver immédiatement et abondamment avec de l’eau et consulter un spécialiste ;
- S28 après contact avec la peau, se laver immédiatement et abondamment avec de l’eau ;
- S36/37/39 porter un vêtement de protection
approprié, des gants et un appareil de protection des
yeux et du visage.
2. Combustion du carbone
L’équation chimique de la réaction est :
C + O2 → CO2
3. Combustion du dihydrogène
L’équation chimique de la réaction est :
2 H2 + O2 → 2 H2O
CHAPITRE 13 - Que se passe-t-il lorsque des produits d’usage courant réagissent entre eux ? • 51
4. L’acide chlorhydrique dissout le tartre
c. L’acide contenu dans ce produit est de l’acide
L’équation chimique de la réaction est :
chlorhydrique.
2 (H+ + Cl–) + CaCO3 → (Ca2+ + 2Cl–) + H2O + CO2
5. L’acide chlorhydrique décape le fer
L’équation chimique de la réaction est :
2 (H+ + Cl–) + Fe → (Fe2+ + 2Cl–) + H2
Appliquer le cours
6. Écrire des équations chimiques avec des
coefficients convenables
a. 3 Fe + 2 O2 → Fe3O4
9. Écrire les équations bilan
a. N2 + 3 H2 → 2 NH3
b. 2 Al + Fe2O3 → 2 Fe + Al2O3
c. 2 Na + Cl2 → 2 NaCl
d. 2 C2H2 + 5 O2 → 4 CO2 + 2 H2O
e. 4 Al + 3 O2 → 2 Al2O3
f. 2 Cu + O2 → 2 CuO
g. Ca + 2 H2O → Ca(OH)2 + H2
b. 2 Al + Fe2O3 → 2 Fe + Al2O3
h. Fe2O3 + 3 CO → 2 Fe + 3 CO2
c. CH4 + 2 O2 → CO2 + 2 H2O
10. Conserver les atomes et les charges
7. Chercher l’erreur.
a. Zn + 2 (H+ + Cl–) → (Zn2+ + 2 Cl–) + H2
a. L’élève ne porte pas de blouse, ni de gants.
b. L’élève pipette le liquide avec la bouche, ce qui
b. Cu2+ + Fe → Cu + Fe2+
est formellement interdit. Il doit utiliser une poire à
pipeter.
c. Les bouchons ou les compte-gouttes ne doivent
pas rester sur la paillasse. Il faut reboucher les flacons
de produits chimiques après utilisation.
d. L’élève doit manipuler l’acide concentré avec des
gants.
Il faut toujours verser l’acide concentré dans l’eau et
non le contraire. En effet, le dégagement de chaleur
qui se produit au contact de l’eau et de l’acide peut
provoquer des projections d’acide.
e. L’élève n’a pas gardé les lunettes sur les yeux pour
manipuler l’acide.
f. L’élève ne doit pas garder les gants pour allumer
des allumettes. Il doit fermer la boîte avant de frotter
l’allumette.
8. Précautions d’emploi
a. Code des risques encourus :
Provoque de graves brûlures : R 34
b. Code des moyens de sécurité à mettre en œuvre :
- S1/2 : conserver sous clé et hors de portée des
enfants ;
- S 24 et S 25 : éviter le contact avec la peau et les
yeux ;
- S 26 : en cas de contact avec les yeux, laver immédiatement et abondamment avec de l’eau et consulter un spécialiste ;
- S28 : après contact avec la peau, se laver immédiatement et abondamment avec de l’eau et du savon ;
- S36/37/39 : porter un vêtement de protection
approprié, des gants et un appareil de protection des
yeux et du visage ;
- S 45 : en cas d’accident ou de malaise, consulter
immédiatement un médecin (si possible lui montrer
l’étiquette).
c. 3 Cu2+ + 2 Al → 3 Cu + 2 Al3+
d. 2 Al + 6 (H+ + Cl–) → 2 (Al3+ + 3 Cl–) + 3 H2
e. ClO– + 2H+ + Cl– → Cl2 + H2O
11. Vérifier la conservation des atomes
a. C3H8 + 5 O2 → 3 CO2 + 4 H2O
b. On trouve trois atomes de carbone, huit atomes
d’hydrogène et dix atomes d’oxygène dans les réactifs
et les produits.
c. Il y a bien conservation des atomes au cours de la
réaction.
12. Vérifier la conservation des atomes
a. Les réactifs mis en présence sont le fer et le dioxygène. Le produit obtenu est l’oxyde de fer.
b. La loi mise en évidence par cette observation est
la loi de Lavoisier : la somme des masses des réactifs
qui réagissent est égale à la somme des masses des
produits formés.
c. L’équation de la réaction est :
3 Fe + 2 O2 → Fe3 O4.
13. Vérifier la conservation de la charge
électrique.
a. Dans les réactifs, les ions positifs sont les ions
hydrogène : 2 H+ et les ions négatifs sont les ions chlorure : 2 Cl–.
b. La charge totale des ions présents dans les réactifs
est nulle : (2+) + (2–) = 0.
c. Dans les produits, les ions positifs sont les ions
magnésium : Mg2+ et les ions négatifs sont les ions
chlorure : 2 Cl–.
d. La charge totale des ions présents dans les produits est nulle : (2+) + (2–) = 0.
e. La conservation de la charge électrique est satisfaite.
52 • CHAPITRE 13 - Que se passe-t-il lorsque des produits d’usage courant réagissent entre eux ?
Utiliser ses connaissances
d. Masse de minerai nécessaire pour obtenir
14. Équation chimique et bilan molaire
11,2 tonnes de fer :
11,2 tonnes de fer représentent :
a. Les réactifs de la réaction chimique sont l’éthanol
de formule C2H6O et le dioxygène de l’air de formule
O2. Les réactifs sont le dioxyde de carbone de formule
CO2 et l’eau de formule H2O.
b. L’équation de la réaction est :
C2H6O + 3 O2 → 2 CO2 + 3 H2O
c.
Équation
C2H6O + 3 O2
bilan
Bilan
1 mol 3 mol
molaire
→
2 CO2 + 3 H2O
2 mol
3 mol
d. Bilan molaire de la réaction : une mole d’éthanol
réagit avec trois moles de dioxygène pour former deux moles de dioxyde de carbone et trois moles
d’eau.
15. Combustion du fer
a. L’équation de la réaction s’écrit :
3 Fe + 2 O2 → Fe3O4
b. Bilan molaire de la réaction : trois moles de fer
réagissent avec deux moles de dioxygène pour former
une mole d’oxyde de fer.
c. Nombre de moles de fer qui ont réagi
Équation
bilan
Bilan
molaire
Nombre
de moles
3 Fe
3 mol
+  2 O2 →
Fe3O4
2 mol
n
1 mol
11,2
× 106 = 2 × 105 moles de fer.
56
Équation
Fe2O3 + 3 CO bilan
Bilan
1 mol 3 mol
molaire
Nombre
n
de moles
0, 2 × 3
= 0,6 mol.
1
d. Masse de fer qui a réagi : 0,6 × 56 = 33,6 g.
Masse molaire de l’oxyde : M = 3 × 56 + 4 × 16
= 232 g/mol.
Masse d’oxyde formé : 0,2 × 232 = 46,4 g
16. Élaboration de l’acier
a. Équation chimique de la réaction :
Fe2O3 + 3 CO → 2 Fe + 3 CO2
2 Fe +  3 CO2
2 mol
3 mol
2 × 105
mol
Le nombre n de moles d’oxyde de fer utilisé est
105 moles.
Masse de minerai : 160 × 105 = 16 × 106 g = 16 tonnes.
17. Dilution de l’acide chlorhydrique
30
= 1,5 L.
100
70
Volume d’eau à mélanger : 5 ×
= 3,5 L.
100
b. Il faut verser en premier l’eau dans le seau, car on
doit toujours verser l’acide dans l’eau.
c. Après traitement, il faut rincer le seau abondamment.
a. Volume d’acide à mélanger : 5 ×
18. Un peu de mathématiques
a. Le pictogramme signifie que le produit est irritant.
b. La masse d’un litre de solution est :
1,12 × 1 000 = 1 120 g.
c. Masse d’acide chlorhydrique contenu dans un
litre de solution :
23
× 1 120 = 257,6 g.
100
0,2 mol
n = 
→
19. Le chalumeau oxyacétylénique
a. La molécule d’acétylène de formule C2H2 est composée d’atomes de carbone et d’hydrogène.
b. L’équation de la réaction est :
2 C2H2 + 5 O2 → 4 CO2 + 2 H2O
c. L’équation de la combustion du fer dans le dioxygène est :
3 Fe + 2 O2 → Fe3O4
d. Si on brûle 6 moles de fer, il apparaît 2 moles de
Fe3O4.
b. Bilan molaire de la réaction : une mole d’oxyde
e. La masse de fer qui a réagi est :
de fer réagit avec trois moles de monoxyde de carbone pour former deux moles de fer et trois moles de
dioxyde de carbone.
c. Masse molaire de Fe2O3 :
M = 2 × 56 + 3 × 16 = 160 g/mol.
6 × 56 = 336 g.
Masse molaire de Fe3O4 : M = 3 × 56 + 4 × 16
= 232 g/mol.
La masse d’oxyde qui est apparue est :
2 × 232 = 464 g.
CHAPITRE 13 - Que se passe-t-il lorsque des produits d’usage courant réagissent entre eux ? • 53
14 Comment préserver son audition ? HS3
Les activités
■■Activité 1Comment évolue le niveau d’intensité acoustique lorsque
le récepteur s’éloigne de la source sonore ?
•Matériel
Un GBF - Un haut-parleur - Un interrupteur - Un sonomètre - Une règle
Distance d entre le haut-parleur et le sonomètre (cm)
Niveau sonore L en dB (arrondi à l’unité)
5
80
10
74
20
68
40
62
•Réponses aux questions
1. Voir figure ci-contre.
L (dB)
80
75
2. Le niveau d’intensité acoustique diminue lorsque
70
la distance entre la source et le récepteur augmente.
Cette diminution est de 6 décibels chaque fois que la
distance double.
65
3. S’éloigner de la source sonore constitue une bonne
protection contre les sons.
60
d (cm)
0
0
5
10
20
40
■■Activité 2 Comment protéger son oreille ?
•Matériel
Un GBF - Un haut-parleur - Un interrupteur - Un sonomètre
Des PICB (protections individuelles contre les bruits)
•Réponses aux questions
1. 2. 3. Les mesures et les réponses aux questions dépendent des PICB choisis.
4. L’utilisation de dispositifs de protection individuelle contre le bruit constitue une protection
efficace contre le bruit.
Les documents
■■Le diagramme de Fletcher et Munson
•Réponses aux questions
1. Non, ces sons ne sont pas obligatoirement perçus de la même façon. Un son de 40 décibels et
5 000 hertz est perçu beaucoup plus fort qu’un son de 40 décibels et 100 hertz.
2. Une variation de 10 décibels produit plus d’effets avec un son de 100 hertz qu’avec un son de
3 000 hertz. Pour un son de 100 hertz, à partir du seuil d’audition, une variation de 10 décibels permet
de couper quatre courbes isosoniques alors que pour un son de 3 000 hertz, on ne coupe que deux
courbes.
54 • CHAPITRE 14 - Comment préserver son audition ?
3. Pour un son de 4 000 hertz, la douleur apparaît pour un niveau d’intensité sonore d’environ
113 décibels.
4. L’oreille humaine est plus sensible aux sons dont la fréquence est comprise entre 2 000 et
5 000 hertz.
■■La prévention contre les traumatismes auditifs
•Réponses aux questions
1. Deux amis assistent à un concert. Un seul d’entre eux utilise des PICB.
2. Le PICB utilisé dans la BD est un bouchon d’oreille.
3. Juste après le concert, la personne qui ne s’est pas protégée a les oreilles qui sifflent.
4. Le médecin diagnostique des acouphènes. Ce sont des sifflements permanents.
5. Sur l’affiche, on peut lire « Un seul concert peut vous ruiner la vie. » donc une seule exposition à un
niveau sonore élevé peut avoir de graves conséquences.
6. Si une impression de surdité ou des sifflements persistent le lendemain d’un concert, il est
conseillé de consulter un médecin. Cette consultation doit avoir lieu au plus tard dans les 72 heures
après le concert.
Les exercices
Tester ses connaissances
Q.C.M.
1. B ; 5. B ;
2. B, C ;
6. A ;
3. B ; 7. A ; mousse est plus important que pour ceux en cire : les
PICB en mousse sont les plus performants.
4. Comparaison de deux casques
4. B ; 8. B, C.
a. Pour protéger de sons de 250 Hz, le casque nº 917B
est plus adapté.
b. Pour protéger de sons de 1 000 Hz, le casque
nº 924A est plus adapté.
Tester ses capacités
1. L en fonction de d
a. Le niveau d’intensité acoustique L diminue lorsque la distance d entre la source sonore et le récepteur augmente.
b.
Distance de mesure d (m)
Niveau d’intensité
acoustique L (dB)
Appliquer le cours
5. Connaître et respecter les limites
a. L’unité du niveau d’intensité sonore est le décibel.
2
4
16
32
98
92
80
74
2. Vrai – Faux
a. Faux : Le niveau d’intensité acoustique se note L.
b. Faux : Le niveau d’intensité acoustique se mesure
avec un sonomètre.
c. Faux : Le niveau d’intensité acoustique diminue
lorsque la distance croît.
d. Vrai : Le niveau d’intensité acoustique décroît
lorsque la distance augmente.
e. Faux : Le niveau d’intensité acoustique augmente
lorsque la distance diminue.
3. Mousse ou cire
L’affaiblissement moyen provoqué par les PICB en
Attention ! l’unité d’intensité sonore est le W.m-2.
b. À partir de 85 décibels il existe un risque auditif.
c. Pour un baladeur à pleine puissance, la durée
maximale journalière d’écoute est de 20 minutes.
En discothèque, la durée maximale journalière
d’écoute est de 4 minutes.
d. Le bricolage ou la chasse peuvent exposer l’oreille
à un risque avéré pour l’audition.
e. Assister sans protection à un essai moteur ou à
une explosion peut être douloureux.
f. Non, avoir une conversation n’est pas dangereux
pour l’audition.
g. Dans le cas « L = 90 dB pendant 3 h », la norme
n’est pas respectée.
Dans le cas « L = 105 dB pendant 4 minutes », la
norme est respectée.
CHAPITRE 14 - Comment préserver son audition ? • 55
6. Vrai ou faux
a. Incomplet : La fatigue auditive provoque une gêne
mais il n’est pas conseillé de rester longtemps à proximité du chantier.
temporaire ou des acouphènes qui sont des sifflements permanents.
b. Faux : Les acouphènes sont des sifflements permanents entendus après une exposition à un bruit
excessif.
c. Vrai : La guérison d’un traumatisme auditif est
impossible.
d. Faux : Un déficit auditif peut n’être que temporaire si l’exposition n’a pas été trop importante.
11. Où se placer ?
7. Reculer pour mieux se protéger
a. L’objectif de la loi est de « protéger les personnes
Hugo mesurera 97 décibels à 4 mètres, 91 décibels à 8
mètres et 85 décibels à 16 mètres.
8. Mettre de l’ordre
Mesure L4 au point A, mesure L2 au point B, mesure L3
au point C et mesure L1 au point D.
9. Choisir le dispositif adapté
a. Pour protéger d’un son de 250 Hz, le casque 2 est
le plus adapté.
b. Pour protéger d’un son de 4 000 Hz, le casque 2 est
le plus adapté.
c. Pour les sons de fréquence 700 Hz : atténuation
avec le casque 1 : 30 dB, atténuation avec le casque 2 :
30 dB.
Pour les sons de fréquence 2 500 Hz : atténuation
avec le casque 1 : 40 dB, atténuation avec le casque
2 : 35 dB.
d. La gamme de fréquences pour laquelle le dispositif 1 est le plus adapté est [700 Hz ; 3 300 Hz].
Utiliser ses connaissances
10. S’éloigner pour se protéger (TICE)
a.
a. Le niveau d’intensité maximale pouvant être
mesuré lors du concert, à un mètre des haut-parleurs,
sans que la norme soit dépassée, est de 140 décibels,
car l’atténuation entre 1 m et 10 m est de 20 dB.
b. L’atténuation sera alors de 32,5 décibels. Le son
mesuré à 40 mètres aura un niveau de 97,5 décibels.
12. La loi
contre un niveau excessif de décibels pour la protection des risques d’altération du système auditif ».
b. Le niveau d’intensité sonore moyen fixé par la loi
est de 105 décibels.
c. Le niveau d’intensité sonore à ne pas dépasser qui
a été fixé par la loi est de 120 décibels.
d. Si un contrôle est effectué, le sonomètre sera placé
au même endroit que le public.
13. Opérateur sur machine
a. L’opérateur peut utiliser un casque ou des bouchons d’oreille.
b. Les dispositifs de l’exercice 3 permettent des affaiblissements supérieurs à 17 décibels pour toutes les
fréquences, donc ils sont suffisants.
14. Se protéger 100 % du temps
a. • Si l’ouvrier enlève ses protections une minute
dans sa journée de travail, la protection effective est
de 23 dB. 100 – 23 = 77 dB. Si le niveau d’intensité
sonore est de 100 dB, alors il ne court pas de risques.
• Si l’ouvrier protège ses oreilles 98 % du temps, la
protection effective est de 17,5 dB. 100 – 17,5 = 82,5 dB.
L’ouvrier est dans une situation où il lui est conseillé
de se protéger pour éviter une fatigue auditive.
• Si l’ouvrier protège ses oreilles 100 % du temps, il
ne risque rien.
b. Si l’ouvrier ne se protège que 50 % du temps, le
niveau d’intensité acoustique ne doit pas dépasser
87 + 3 = 90 dB.
15. Étude d’un audiogramme
a. Audiogramme tenant compte du déficit auditif
(seuil d’audibilité corrigé)
b. Le niveau d’intensité acoustique diminue lorsque
la distance entre la source sonore et le récepteur augmente.
c. Si les passants circulent à moins de quatre mètres
du marteau, ils doivent être protégés. S’ils circulent à
moins de huit mètres, il leur est conseillé de se protéger. Au-delà de huit mètres, il n’y a plus d’obligation
56 • CHAPITRE 14 - Comment préserver son audition ?
140
120
100
80
60
40
20
0
LE CHAMP AUDITIF HUMAIN
(dB)
il de la
Seu
Se
16
siqu
Mu
u il
32
d’a
u
64
d ib
douleur
tral
e orches e
ersation à
n
1
Co v
ilit
é
m
(Hz)
125 250 550 1 000 2 000 4 000 8 000 16 000
b. Non, ce patient ne sera pas gêné pour suivre une
conversation à un mètre.
c. Oui, ce patient est gêné pour suivre un concert de
musique orchestrale : sa perception des sons aigus
est altérée.
16. Situation problème
Les facteurs de risques sont :
- la durée lors de l’écoute (voir graphique du cours,
page 170) ;
- le niveau d’intensité sonore (voir graphique du
cours, page 170) ;
- la hauteur des sons écoutés. Les sons dont la fréquence est proche de 4 000 Hz sont plus dangereux
pour l’audition que les sons plus graves.
15 Comment peut-on décrire le mouvement
d’un véhicule ? T1
Les activités
■■Activité 1 Comment un enregistrement permet-il de connaître
la nature d’un mouvement ?
•Matériel
Un scooter, une bicyclette, une voiturette tirée par un fil
Un caméscope, une webcam reliée à un ordinateur
Un logiciel de mécanique de traitement de l’image : Avistep, Aviméca,…
Une règle ou un repère de 1 mètre (caméscope) ou 10 centimètres (webcam) pour un mobile proche
Expérience
Le professeur peut faire réaliser les films et les traiter en projection à l’aide du logiciel.
Chaque élève peut également traiter les films mis en commun sur un poste informatique individuel.
•Réponse aux questions
1. Le scooter roule à vitesse constante dans le film 2. Le repère mesure 1 m de longueur. Les distances
entre deux croix jaunes consécutives sont de 0,42 m.
1
La vitesse moyenne du scooter est : 0,42 ÷
= 10,5 m/s soit 37,8 km/h.
25
1
2. Le film 3 correspond à un mouvement accéléré : à des intervalles de temps égaux à
de seconde
25
correspondent des distances parcourues de plus en plus grandes.
Le film 1 correspond à un mouvement ralenti : les distances parcourues sont de plus en plus petites.
3. Une chronophotographie ou un film vidéo permettent de déterminer la nature du mouvement
d’un point d’un mobile et de réaliser des mesures et des calculs : distances, durées, vitesses moyennes.
■■Activité 2Le mouvement d’un point d’un objet dépend-il
du référentiel choisi ?
•Matériel
Une bicyclette
Un support d’atelier (on peut aussi renverser la bicyclette et la poser sur la selle et le guidon)
Deux pastilles autocollantes
CHAPITRE 15 - Comment peut-on décrire le mouvement d’un véhicule ? • 57
•Réponse aux questions
Situation 1
1. Le point G (le guidon) se déplace et a pour trajectoire un segment de droite.
2. Le point V (la valve) se déplace et décrit une courbe appelée cycloïde.
Situation 2
3. Le point G est immobile par rapport au sol et par rapport à la bicyclette.
4. Le point V se déplace par rapport au sol et par rapport à la bicyclette : sa trajectoire est un cercle
centré sur le moyeu de la roue.
5. La trajectoire d’un point d’un objet en mouvement dépend du référentiel choisi. Ainsi, le
mouvement de la valve d’une roue se déplaçant sur le sol est une cycloïde si le référentiel choisi est le
sol et un cercle si le référentiel choisi est le cadre de la bicyclette.
Les documents
■■Analyse d’un film vidéo
•Réponse aux questions
1
s ou 0,04 s.
25
2. Le référentiel généralement choisi est tout objet immobile sur le sol de la terre (référentiel
terrestre).
1. Sur un caméscope, deux images successives sont séparées par
3. Pour calculer la vitesse moyenne du mobile, il faut réaliser l’étalonnage d’un objet de dimension
connue, puis placer des croix colorées sur les différentes positions prises par un point du mobile.
On peut aussi faire calculer les vitesses moyennes par le logiciel après avoir choisi les axes du repère.
■■Distances de sécurité
•Réponse aux questions
1. Facteurs influant sur la distance de freinage :
- l’état du véhicule ;
- l’état de la chaussée ;
- la vitesse du véhicule.
2. Lorsque la vitesse est multipliée par 2, la distance de freinage est multipliée par 4 ; lorsque la vitesse
est multipliée par 4, la distance de freinage est multipliée par 16.
Ces résultats montrent que la distance de freinage dF est proportionnelle au carré de la vitesse v du
véhicule. On a dF = k.v2.
3. Pour une même vitesse du véhicule, la distance de freinage double sur route mouillée,.
4. Vitesse de la voiture exprimée en m/s :
v = 130 ÷ 3,6 ≈ 36,1 m/s .
Lors du temps de réaction, le conducteur parcourt en 1,2 seconde :
36,1 × 1,2 = 43 m.
D’où une distance d’arrêt sur route sèche :
dA = dR + dF = 43 + 95 = 138 m.
Sur route mouillée : d’A = 43 + 190 = 233 m.
58 • CHAPITRE 15 - Comment peut-on décrire le mouvement d’un véhicule ?
Les exercices
Tester ses connaissances
Q.C.M.
1 : C ; 2 : B et C ; 3 : A et C ; 4 : B et C ; 5 : B.
Tester ses capacités
b. La distance entre Pau et Lannemezan par le rail est
de 78 km.
Vitesses moyennes :
TER : v m oyenne
1. Ascenseur
78
≈ 1,083 km/min ou 65 km/h.
72
78
=
≈ 0,716 km/min ou 43 km/h.
109
Corail : v m oyenne =
Référentiels possibles : le palier et Hubert.
10. Reconnaître un mouvement
2. Scooter et bicyclette
Mouvement uniforme en 2 ; accéléré en 1 et 3 ; uniformément accéléré en 1 ; uniformément ralenti en 4.
a. Référentiel choisi : la route.
b. Référentiel choisi : la route.
11. Déplacement d’une voiture
a. Tableau
3. Escalier mécanique
a. Paul et Nora sont en mouvement par rapport au
sol.
Jim est en mouvement par rapport au tapis roulant.
b. Jim est immobile par rapport au sol. Paul et Nora
sont immobiles par rapport au tapis roulant.
Date (s)
Vitesse
(km/h)
Vitesse
(m/s)
0
1
2
3
4
5
72
81
89
98
106
116
20
22,5
24,7
27,2
29,4
32,2
b. Représentation graphique de la vitesse en fonc-
4. Trajectoires de billes
Trajectoires : rectiligne en 1 ; circulaire en 3 ; quelconque en 2.
tion de la durée :
Vitesse (m/s)
32
5. Lancer de marteau
a. La main tient le câble : la trajectoire de la boule est
circulaire de M1 à M8.
b. La main a lâché le câble : la trajectoire de la boule
est rectiligne de M9 à M12.
20
6. Chronophotographie
a. Mouvement uniforme en 1.
b. Mouvement ralenti en 3.
c. Mouvement accéléré en 2.
4
date (s)
0
1
2
3
4
5
c. La vitesse est une fonction affine du temps : le
Appliquer le cours
mouvement de la voiture est uniformément accéléré.
7. Unités de temps
1
h = 0,017 h.
60
1
1
b. 15 min = h = 0,25 h ; 20 min = h ≈ 0,33 h ;
4
3
1
3
30 min = h = 0,5 h ; 45 min = h = 0,75 h .
2
4
a. 1 h = 60 min ; 1 min = 
8. Convertir rapidement
Utiliser ses connaissances
12. Porte-outil automatisé
a. Positions du porte-pipette
b.
y (cm)
20
a. 110 km/h = 30,6 m/s.
b. 10 m/s = 36 km/h.
10
9. Horaires SNCF
4
13
5
6
7
3
9
1,2
10,11
a. Corail : durée du trajet : 1 h 12 min ; TER : durée du
trajet : 1 h 49 min.
Le trajet le plus rapide est celui du Corail.
8,12
x (cm)
0
10
20
30
40
CHAPITRE 15 - Comment peut-on décrire le mouvement d’un véhicule ? • 59
c. Les phases de repos sont : 1 à 2 ; puis 10 à 11. La
15. Distance de sécurité
phase de mouvement rectiligne uniforme concerne
les positions : 4 à 8.
a. Recommandation à respecter : laisser deux traits
Lors de cette phase, la vitesse du porte-pipette est
de :
v=
40
= 20 cm/s .
4 × 0,5
13. Étude d’un freinage
a. Origine des temps et des espaces : la position du
motocycliste au début du freinage.
b. Le freinage dure 6 secondes : la position atteinte
est de 108 m.
c. Le calcul des vitesses moyennes se calcule ainsi :
En C3 : = (D2-B2)/(D1-B1) ; on sélectionne la zone :
C3 : M3, puis on active la fonction : recopier à droite.
entre deux véhicules.
130 km/h = 36,1 m/s. Donc 72 m sont parcourus en
2 secondes.
b. Entre deux bandes blanches, il y a :
38 + 14 + 38 = 90 m.
c. Cette distance est supérieure à 72 m : la sécurité
est respectée.
16. Fête foraine
a. L’âne et le lama décrivent des trajectoires circulaires.
b. Longueur d’un tour :
pour l’âne : L = π.D = π × 3 = 9,4 m ;
pour le lama : L = π . D = π × 6 = 18,8 m.
c. Vitesses moyennes des deux animaux :
9,4
≈ 0,52 m/s ;
18
18,8
le lama : v A =
≈ 1,04 m/s .
18
Le lama se déplace deux fois plus vite que l’âne.
l’âne : v A =
17. Parachutisme
a. Représentation graphique :
Distance (m)
d. À l’instant t = 0, v = 37 m/s soit 133 km/h.
100
La moto est immobile pour t = 6 s.
Le mouvement de la moto, lors de cette phase, est
uniformément ralenti, car la vitesse v est une fonction affine du temps t.
50
14. Distances de freinage
a. La distance de freinage dépend de :
- l’état de la route ;
- l’état de la voiture ;
- la vitesse de la voiture.
b. Lorsque la vitesse double, la distance de freinage
est multipliée par 4 ; Lorsque la vitesse triple, la distance de freinage est multipliée par 9.
c. Sur route sèche :
CS =
v2
33,32
=
≈ 0,786 .
2g .DF 2 × 9,8 × 72
Sur route mouillée, CS est deux fois plus faible :
CS = 0,393.
10
0
temps (s)
1
2
3
4
5
b. Sur des intervalles de temps égaux, les distances
parcourues ne sont pas égales ; elles augmentent avec
le temps. Le mouvement n’est pas uniforme, mais
accéléré.
c. Pour t = 5 s et x = 121 m, on a :
2 ×121
=
= 9,7 m/s2.
t2
52
Cette valeur est voisine de 9,8 m/s2.
a=
2x
60 • CHAPITRE 15 - Comment peut-on décrire le mouvement d’un véhicule ?
16 Comment passer de la vitesse des roues
à celle de la voiture ? T2
Les activités
■■Activité 1 Que mesure un tachymètre ?
•Matériel
Une platine
Un moteur entraînant un disque translucide marqué d’une bande noire
Un générateur continu 6/12V et un potentiomètre électronique
Un tachymètre par contact ou par réflexion
Un montage de photocoupleur (fourche électronique)
Des fils de connexion
•Montage du photocoupleur
Un photocoupleur appelé aussi fourche optoélectronique est une association d’un émetteur
(DEL infrarouge) et d’un détecteur (phototransistor ou photodiode).
Pour réaliser l’expérience, on peut le brancher ainsi :
+4à6V
470 Ω
4,7 kΩ
YS
E
D
M
•Réponse aux questions
Expérience 1
1. Les pics correspondent aux passages de la bande noire devant le capteur du photocoupleur.
Notre disque comportait 6 bandes noires régulièrement espacées.
2. Dans notre expérience, la période du disque : T = 120 ms soit 0,12 s.
3. La fréquence de rotation du disque : n = 8,3 tr/s.
Expérience 2
4. Le tachymètre mesure la fréquence de passage d’une bande noire : 3 066 tr/min. Donc une
fréquence de rotation du disque :
3 066 ÷ 6 = 511 tr/min.
5. Pour passer des tours par minute aux tours par seconde, on divise par 60 :
511 ÷ 60 ≈ 8,5 tr/s .
6. On retrouve à peu près la fréquence précédente : 8,3 tr/s.
CHAPITRE 16 - Comment passer de la vitesse des roues à celle de la voiture ? • 61
■■Activité 2Quelle est la relation entre la fréquence de rotation
et la vitesse linéaire ?
•Matériel
Une roue de bicyclette sur un support - Un compteur de bicyclette (cyclomètre)
Un tachymètre - Un mètre pliant
•Résultats
Pour la roue : R = 33,5 cm d’où une circonférence L = 210 cm.
1re allure
2e allure
3e allure
v (km/h)
6,7
11
18
v (m/s)
1,86
3,05
5,0
N (tr/min)
56
90
148
n (tr/s)
0,93
1,50
2,47
2π.R.n
1,95
3,15
5,19
•Réponse aux questions
1. L’expression 2 π.R correspond à la circonférence de la roue.
2. Pour chacune des trois allures, on a approximativement (les deux colonnes grisées) l’égalité :
v = 2 π.R.n.
3. La relation entre la fréquence de rotation n d’une roue et la vitesse v de la bicyclette est :
v = 2 π.R .n.
Les documents
■■La transmission du mouvement circulaire
•Réponses aux questions
1. Une bicyclette utilise la transmission par chaîne ; une voiture, certaines motos utilisent la
transmission par pignons.
2. De n1 . D1 = n2 . D2, on sort : n2 =
n1 .D1 1500 × 250
=
= 3750 tr/min .
D2
100
3. Le rapport de transmission est : RT =
nsortie
nentrée
=
3750
= 2,5 .
1500
Lorsque la poulie motrice effectue 1 tour, la poulie réceptrice effectue 2,5 tours.
■■La transmission de la moto Kawasaki ER-6
•Réponses aux questions
1. Pour un régime moteur n1 = 6 600 tr/min (en 4e) :
- pour la transmission : {moteur / arbre primaire} :
n .Z
6600 × 42
n1 . Z1 = n2 . Z2 d’où : n2 = 1 1 =
= 3150 tr/min
Z2
88
- pour la transmission : {arbre secondaire / arbre primaire} :
n3 . Z3 = n4 . Z4 d’où : n4 =
n3 .Z 3
=
Z4
3150 × 27
= 2835 tr/min
30
- pour la transmission : {roue arrière / arbre secondaire} :
n5 . Z5 = n6 . Z6 d’où : n6 =
2. Le rapport de transmission total est : RT =
technique du fabricant.
n5 .Z 5
Z6
nsortie
nentrée
=
=
2835 ×15
= 924 tr/min
46
924
= 0,1400 . On retrouve la valeur de la fiche
6600
62 • CHAPITRE 16 - Comment passer de la vitesse des roues à celle de la voiture ?
Les exercices
Tester ses connaissances
Q.C.M.
1 : A et B ; 2 : A et B ; 3 : A et C ; 4 : B ; 5 : B et C ; 6 : A et B.
- la durée pour effectuer un tour s’appelle la période ;
- la période T d’une rotation est liée à la fréquence
1
n par la relation n = ; dans cette relation, T s’exT
prime en seconde et n en tour par seconde.
9. Calculs de période et de fréquence
Tester ses capacités
1. Lecture d’une fréquence de rotation
a. L’appareil est un tachymètre.
b. Il est gradué en tour par minute.
c. La fréquence de rotation est de :
35 × 100 = 3 500 tr/min.
Période
T (s)
1
2
0,01
4 × 10-4
0,2
40
Fréquence
de rotation
n (tr/s)
1
0,5
100
2 500
5
0,025
10. Vitesse de rotation
2. Expérience
a. Entre deux pics : 40 ms.
b. Un tour de disque dure 40 ms ou 0,04 s. 
D’où la fréquence de rotation : n =
1
1
=
= 25 tr/s .
T 0,04
3. Fréquencemètre de vélo
a. Le compteur affiche des tours par minute : RPM
signifie « Rotation Per Minute » en anglais.
b. n = 84 tr/min.
4. Mesures de fréquence de rotation et de
vitesse linéaire
a. Vitesse du cycliste : 41,7 km/h. Fréquence de rotation des roues : 321 tr/min.
b. Vitesse : 11,58 m/s ; fréquence : 5,35 tr/s.
c. π.D.n = π × 0,69 × 5,35 = 11,60 m/s. On retrouve
pratiquement la vitesse du cycliste affichée par le
compteur.
5. Termes d’une expression
a. v : vitesse linéaire de l’outil, R : rayon de l’outil ;
n : fréquence de rotation.
b. Unités : v en mètre par seconde, R en mètre et n en
tour par seconde.
6. Tondeuse électrique
a. N = 2 790 ÷ 60 = 46,5 tr/s.
b. V = π.D.n = π × 0,38 × 46,5 = 55,5 m/s.
7. Scie circulaire portative
v
30
a. Pour v = 30 m/s, n =
=
= 50, 3 tr/s .
π .D π × 0,19
b. Soit environ 3 000 tr/min.
a. n = 50 tr/s.
b. n = 3 000 tr/min.
11. Cyclomètre
a. L’écriture correcte de l’unité de vitesse : km/h.
b. 18 km/h = 5 m/s.
c. n =
v
5
=
= 2, 27 tr/s . ou 136 tr/min.
2π .R 2π × 0, 35
12. Grandeurs et unités
Grandeur
physique
Symbole de la
Symbole de
Unité légale
grandeur
l’unité
T
seconde
s
Fréquence de
rotation
Période
n
tour par
seconde
tr/s
Vitesse linéaire
v
mètre par
seconde
m/s
13. Calculs
Objet en
rotation
v
R
n
Lame de scie
circulaire
50,3 m/s
12 cm
4 000 tr/min
Pale
d’éolienne
9,4 m/s
15 m
6 tr/min
Fraise de
Dremel
13 m/s
3,9 ×
ou 3,9 mm
32 000 tr/min
Roue de moto
120 km/h
0,38 m
14tr/s ou 840
tr/min
Satellite SPOT
II
7 550 m/s
4 030 km
2,98 × 10-4 tr/s
ou ≈ 1 tr/h
10-3 m
14. Pour lire l’heure
a. Les périodes : aiguille des heures : 12 h pour
8. Période et fréquence
Lors du mouvement de rotation d’un objet autour
d’un axe :
- les trajectoires de chaque point de l’objet sont des
cercles ;
- l’objet effectue un mouvement uniforme si sa fréquence de rotation est constante ;
1 tour ; aiguille des minutes : 1 h pour 1 tour ; aiguille
des secondes : 1 min pour 1 tour.
b. Les fréquences : respectivement :
1
tr/min = 1,39 × 10-3 tr/min,
720
1
tr/min = 0, 017 tr/min et 1 tr/min .
60
CHAPITRE 16 - Comment passer de la vitesse des roues à celle de la voiture ? • 63
15. Satellites terrestres artificiels
18. Scooter
1
a. Météosat : fréquence de rotation :
tr/s ;
86 400
a. Arbre moteur : n1 = 6 600 tr/min
1
vitesse linéaire : v = 2π.R.n = 2π × 4,2 × 107 ×
86 400
Variateur :
n2
D1 = 70 mm
6 600 × 90
n2 =
= 8486 tr/min .
70
b. Le rapport de transmission :
Z
Z
13 12
RT = 1 x 3 =
×
= 0, 068 2 .
Z2
Z 4 52 44
= 3 054 m/s ou 11 000 km/h.
b. La Station Spatiale Internationale (ISS) :
v = 27 700 km/h = 7 700 m/s.
Circonférence de l’orbite : L = 2π.R = 4,25 × 107 m
soit 42 500 km.
Fréquence de rotation :
n=
v
7700
=
= 1,81×10 −4 tr/s
2π .R 4,25 ×107
soit une période de 5 524 s ou 92 min.
16. Perçage dans l’acier
a. 270 tr/min = 4,5 tr/s.
b. Vitesse de coupe du foret : v = π . D . n
= π × 28 ×10-3 × 4,5 = 0,396 m/s soit 23,8 m/min.
c. Nombre de tours de foret pour avancer de 18 mm :
18 ÷ 0,23 = 78,2 arrondis à 79 tours.
Donc une durée de : 79 ÷ 4,5 = 17,6 s.
17. Le choix de la vitesse
a. Circonférence d’une roue : π.D = π × 0,69 = 2,168 m
au millimètre près.
b. Vitesse : v = 36 km/h soit 10 m/s. La durée correspondant à une longueur de roue sur le sol est de :
2,168 ÷ 10 = 0,2168 s.
c. La période de la roue en rotation est donc
T = 0,2168 s soit une fréquence de :
n=
1
= 4,61 tr/s ou 277 tr/min.
0,216 8
D1 = 90 mm
Lorsque le variateur effectue 1 tour, la roue arrière
effectue 0,068 2 tour.
Lorsque l’arbre moteur fait 1 tour, le variateur effec90
tour et la roue arrière fait :
tue
70
90
× 0, 0682 = 0, 0877 tour .
70
À 6 600 tr/min, la roue tourne à la fréquence de
6 600 × 0,087 7 = 579 tr/min, soit à la vitesse de :
579 × π.D = 691 m/min ou 41,5 km/h.
La vitesse maximale de 45 km/h est bien respectée.
19. Lecteur de CD
a. Pour v = 1,2 m/s, la durée pour un déplacement
sur une alvéole de 2,224 × 10-6 m est :
Δt = 2,224 ×10 −6 ÷ 1,2 = 1,85 ×10 −6 s .
b. Sur la première piste : R = 22 × 10-3 m et
n=
1, 2
v
=
= 8, 68 tr/s .
2π .R 2π × 22 × 10 −3
Sur la dernière piste : R = 58 × 10-3 m et
1, 2
v
n=
=
= 3, 29 tr/s .
2π .R 2π × 58 × 10 −3
Selon la position de la tête de lecture sur le disque, la
fréquence de rotation du moteur varie.
d. Roue :n2 = 277 tr/min
20. Démarche d’investigation
Un spectateur de la grande roue doit :
- mesurer le diamètre approximatif de la grande
roue en imaginant sa projection sur le sol (avec un
mètre pliant) ;
- chronométrer la durée mise par une nacelle pour
faire un tour, à l’aide du chronomètre d’une montre
électronique ;
- utiliser la relation v = π.D.n, la fréquence de rotation n étant déduite de la période T précédemment
mesurée.
D2
Pédalier : n1 = 80 tr/min à 100 tr/min
D1 = 50 dents
Pour n1 = 80 tr/min : D2 =
80 × 50
= 14,4 .
277
Pour n1 = 100 tr/min : D2 =
100 × 50
= 18,05 .
277
Pour rouler à 36 km/h avec ces fréquences de pédalage, il peut utiliser les pignons de 15, 16 et 18 dents.
Téléchargement